Sie sind auf Seite 1von 19

TOPIC: OWNERSHIP

Manantan vs. Aniceto Somera;


GR No. 145867 April 07, 2009

Facts: On 10 March 1998, Soledad Manantan filed with the MTCC, a Complaint for ejectment and damages against respondent Aniceto Somera and a
certain Presentacion Tavera. That when she caused the relocation survey of her said property above-mentioned, she discovered that the respondent
and Tavera had occupied portions thereof, by reason of which she called their attention with a request that they vacate their respective areas as soon
as she would have need of the same, or when she decides to sell the same to any interested buyer; That only recently, she wanted to sell her property
above-mentioned to an interested buyer, but that upon knowing of the respondent and Taveras encroachments, the prospective buyer decided not to
proceed with the sale until after the property shall have been first vacated by the respondent and Tavera; That she asked the them to vacate her
property, but that they refused to do so, and that after making more demands which were all ignored by the respondent and Taveras, petitioner was
forced to consult her lawyer, who immediately wrote them a final formal demand to vacate her land, but to no avail;

Issue:
1. WON THE MUNICIPAL TRIAL COURT IN CITIES, BAGUIO CITY, BRANCH 1, HAD THE JURISDICTION OVER THE ACTION - EJECTMENT AND
DAMAGES ENTITLEDSOLEDAD MANANTAN, PLAINTIFF, V. ANICETA SOMERA AND PRESENTACION TAVERA, AND ALL PERSONS CLAIMING
RIGHTS UNDER THEM, DEFENDANTS;
2. WON the action for ejectment would prosper by the petitioner

Ruling:
1. In the main, petitioner argues that the Complaint is in the nature of an action for unlawful detainer over which the MTCC had jurisdiction. An
action for forcible entry or unlawful detainer is governed by Rule 70 of the Rules of Court where it states that .... person against whom
the possession of any land or building is unlawfully withheld after the expiration or termination of the right to hold
possession by virtue of any contract, express or implied, or the legal representatives or assigns of any such lessor, vendor,
vendee, or other person, may, at any time within one (1) year after such unlawful deprivation or withholding of possession,
bring an action in the proper Municipal Trial Court Noticeably, the Complaint does not allege facts showing compliance with
the prescribed one year period to file an action for unlawful detainer. It does not state the material dates that would have established that
it was filed within one year from the date of Manantans last demand upon respondent to vacate the disputed portion of land. Such allegations
are jurisdictional and crucial, because if the complaint was filed beyond the prescribed one year period, then it cannot properly qualify as an
action for unlawful detainer over which the MTCC can exercise jurisdiction. It may be an accion publiciana or accion reivindicatoria. Since the
Complaint in Civil Case No. 10467 failed to satisfy on its face the jurisdictional requirements for an action for unlawful detainer, the Court of
Appeals was correct in holding that the MTCC had no jurisdiction over the said Complaint and should have dismissed the
same. There is no possible argument around the lack of jurisdiction of MTCC over Civil Case No. 10467.
2. It bears to stress that Manantans Complaint is dismissed herein for its defects, i.e., its failure to allege vital facts in an action for unlawful
detainer over which the MTCC has jurisdiction.

CASILANG, SR. vs. CASILANG-DIZON ;


G.R. No. 180269 February 20, 2013

FACTS:
The late spouses Liborio and Francisca had eight children. Liborio died intestate on October11, 1982 at the age of 83, followed not long after b his wife
Francisca on 1982.Their son Bonifacio also died while son & Ireneo died on June 11, 1992, survived by his four children, herein respondents. The estate
of Liborio, which left no debts, consisted of three parcels of land located in Calasiao, Pangasinan namely: 1. Lot No. 4676; Lot No. 4704; and Lot 4618.
On May 26, 1997, respondent Rosario filed with the the MTC a complaint for unlawful detainer to e v i c t h er u n c l e, p et it i on er Jo s e , fr o m lo t
4 6 1 8 wa s ow n e d by h er fa t h er I r en eo , a s ev id en c e d by T a x De c l a r a t io n n o. 5 5 5 is s u ed i n 1 9 9 4 u n d er h er fa t h e r s n am e. O n
A pr il 3 , 1 9 9 7 t h e h e ir s o f I r e n e o ex e c u t ed a Deed of Extrajudicial Partition with Quticlaim hereby they adjudicated the LOT no. 4618 to
themselves and renounced their respective shares in Lot. No 4618 in favor of Rosario. Petitioner Jose raised the defense that he was the lawful,
absolute, exclusive owner and in actual possession of the said lot and that he acquired the same through intestate succession from his late
father,Liborio. On February 18, 1998. The MTC rendered judgement finding Rosario to be the owner of Lot No. 4628 and ordering Jose to remove his
house and vacate Lot No. 4618.

O n Ju n e , 1 9 8 8 , t h e pe t i t i on er s , t h e h ei r s o f L i bo r i o a n d F r a n c i s c a , f i le d wi t h t h e R T C a c om p la in t f or An n u l me n t o f
D oc u m en t s , O wn er s h ip a n d P ea c e fu l P os s es s io n w it h Da ma ge s a g a i n s t I r en e o s h e ir s , h e r e in r e s p on de n t s . Am o n g t h e
d oc u m en t s s ou gh t t o b e a n n u ll e d w a s t h e 1 9 9 7 De ed o f E x t r a j u di c ia l P a r t i t i on ex ec u t e d b y I r e n e o s c h i ld r e n ov er L o t NO .
4 6 1 8 AS WE L L AS t d n o . 5 5 5 . T he pe ti t i on e r s a l le g e d i n t he ir c om p la i nt th a t a l l 8 ch i ld r e n of L ib o r i o e nte r e d i nt o
a ve r b a l pa r t it i o n o f h is e st a te pu r s u a n t t o w h ic h Jo se wa s a l lo t te d L ot . N o . 4 6 1 8 a s hi s sh a r e . T h at I r en e o n e v e r
c la im e d o wn er s h ip o f L ot n o . 4 6 1 8 n or t o ok p os s es s io n of i t , b ec a u s e h is s h a r e wa s t h e s ou t h we s t er n 1 / 5 PO R T I O N O F L ot
n o. 4 6 7 6 , c on t a in i n g a n a r e a o f 1 , 3 0 8 s q m et er s o f wh ic h h e t o ok ex c lu s iv e p os s es s io n du r i n g h is li f et i me t h at J os e h as
a lw ay s r es id ed i n L ot n o . 4 6 1 8 s in c e c h il dh o o d wh er e h e bu il t h is f a m i ly s s e i c on c r et e h o u s e j u s t a fe w s t ep s a wa y f r om
h is pa r e n t s o ld ba m bo o h u t t h a t h e t o ok i n a n d c a r ed f or h is a g e d p a r en t s i n h is h ou s e u n t il t h e ir d ea t h s in 1 9 8 2 : t h e on e
o f h is c h il dr en h as al s o bu il t a h ou s e on t h e l ot . Jo s e , s a i d t o b e t h e m os t e du c a t ed o f t h e c a s i la n g s i li n g s w h o w o r k ed as
an i n s u r an c e ag en t .

F or h er pa r t , r es p o n d en t R o s a r i on a l le g ed i n h e r a n s we r t h a t s h e is t h e a c t u a l a n d l a w fu l o w n e r o f L ot . N o . 4 6 1 8 h av in g
a c q u i r e d t h e s am e w a y o f a d ee d o f ex t r a j u di c ia l pa r t it io n w i t h q u i t c la im d a t ed a pr il 3 , 1 9 9 7 wh ic h w a s du l y ex e c u t ed
a mo n g I r en eo s h ei r s a n d t h a t h er o wn er s h i p ov er s u bj ec t pr o pe r t y c ou ld b e t r a c ed b a c k t o h er l a t e fa t h e r I r en eo w h i c h
t h e la t t e r in h er it ed by w a y o f in t es t a t e s u c c es s io n fr o m h is de c e a s ed fa t h er L i b or i o. A f t e r a f u l l t r ia l o n t h e m er it s , T h e
R T C r u le d t h at J os e i s t h e l a w f u l o w n e r a n d p o s s e s s o r of L ot . 4 6 1 8 a n d t h a t t h e De ed o f E x t r a j u di c ia l Pa r t it io n w it h

1
Q u it c la i m da t e d A pr il 3 , 1 9 9 7 ex ec u t e d a mo n g I r en eo s h ei r s is n u ll a n d v o id . C A R E V E R S E D T HE R T C DE CI S I O N, r el y i n g
m ai n l y on t h e M T C d ec is i on o n t h e u n la wf u l de t a in er c a s e a ga in s t JO S E .

I S S UE :
1. W/N THE VERBAL PARTITION AMONG THE HEIRS OF LIBORIO, PURSUANT TO WHICH 8 OF HIS CHILDREN RECEIVED HIS OR HER SHARE
OF HIS ESTATE IS VALID? AND THAT LOT NO. 4618 IS JOSES SHARE AND NOT IRENEO AS CLAIMED BY IRENEOS HEIR HEREIN
RESPONDENT ROSARIO?
2. WHETHER OR NOT MTC HAD THE JURISDICTION TO DECIDE CASE?

HELD:

YES, THE PARTIES VERBAL PARTION IS VALID AND HAS BEEN RATIFIED BY THEIR TAKING POSSESION OF THEIR RESPECTIVE SHARES. THE
VALIDITY OF AN ORAL PARTITION IS WELL SETTLED IN VDA. DE ESPINA V. ABAYA WHEREIN IT WAS DECLARED THAT ORAL PARTITION AMONG
THE HEIRS IS VALID.

JOSES POSSESSION OF LOT NO. 4618 UNDER A CLAIM OF OWNERSHIP IS WELL BORNE OUT BY THE RECORDS. IT IS ALSO CONSISTENT WITH THE
CLAIMED VERBAL PARTION WITH HIS SIBLINGS AND FULLY CORROBORATED BY HIS SISTER WHO FURTHER TESTIFIED THAT THEY EACH HAD
TAKEN POSSESSION OF THEIR OWN SHARES AND BUILT THEIR HOUSES THEREON.

A POSSESSOR OF REAL ESTATE PROPERTY IS PRESUMED TO HAVE TILE THERETO UNLESS THE ADVERSE CLAIMANT ESTABLISHES A BETTER RIGHT.
MOREOVER, UNDER ARTICLE 541 OF THE CIVIL CODE, ONE WHO POSSESS IN THE CONCEPT OF OWNER HAS IN HIS FAVOR THE LEGAL
PRESUMPTION THAT HE POSSESSES WITH A JUST TITLE AND HE CANNOT BE OBLIGED TO SHOW OR PROVE IT. SIMILARLY ARTICLE 433 OF THE
CODE PROVIDES THAT ACTUAL POSSESSION UNDER A CLAIM OF OWNERSHIP RAISES A DISPUTABLE PRESUMPTION OF OWNERSHIP. THUS ACTUAL
POSSESSION AND EXERCISE OF DOMINION OVER DEFINITE PORTIONS OF THE PROPERTY IN ACCORDANCE WITH AN ALLEGED PARTITION ARE
CONSIDERED STRONG PROOF OF AN ORAL PARTITION.

MERE TAX DECLARATIONS AND TAX RECEIPTS OF ROSARIO IS NOT A CONCLUSIVE EVIDENCE OF OWNERSHIP. INFERIOR COURTS ARE EMPOWERED
TO RULE ON QUESTION OF OWNERSHIP RAISED BY THE DEFENDANT IN AN EJECTMENT SUIT BUT ONLY TO RESOLVE THE ISSUE OF POSSESSIO;
ITS DETERMINATION IS NOT CONCLUSIVE ON THE ISSUE OF OWNERSHIP.

IT IS WELL TO BE REMINDED OF THE SETTLES DISTINCTION BETWEEN A SUMMARY ACTION OF EJECTMENT AND A PLENARY ACTION FOR
RECOVERY OF POSSESSION AND OR OWNERSHIP OF THE LAND. WHAT REALLY DISTINGUISHES AN ACTION FOR UNLAWFUL DETAINER FROM A
POSSESSORY ACTION (ACCION PUBLICIANA) AND FROM REINVINDICATORY ACTION IS THAT THE FIRST IS LIMITED TO QUESTION OF POSSESSION
DE FACTO. ESPECIALLY JOSE IS THE DIRECT COMPULSORY HEIR AND ROSARIO WAS NOT ABLE TO PROVE OWNERSHIP. ONE WHO ALLEGES SUCH
BEARS THE BURDEN OF PRROF.

Quijano vs Atty. Amante


GR No. 164277 October 8, 2014

Facts:

The petitioner and her siblings, namely: Eliseo, Jose and Gloria, inherited from their father, the late Bibiano Quijano, the parcel of land registered in the
latter's name under Original Certificate of Title (OCT) No. 0-188 of the Registry of Deeds in Cebu City with an area of 15,790 square meters, more or
less. On April 23, 1990, prior to any partition among the heirs, Eliseo sold a portion of his share, measuring 600
square meters, to respondent Atty. Daryll A. Amante. On July 25, 1991, Eliseo, sickly and in need of money, sold an additional 1/3 portion of his share
in the property to the respondent. On September 30, 1992, Fe, Eliseo, Jose and Gloria executed a deed of extrajudicial partition to divide their fathers
estate (consisting of the aforementioned parcel of land) among themselves. The partition resulted in the portions earlier sold by Eliseo to the
respondent being adjudicated to the petitioner instead of to Eliseo.

Due to the petitioners needing her portion that was then occupied by the respondent, she demanded that the latter vacate it. Despite several demands,
the last of which was by the letter dated November 4, 1994, the respondent refused to vacate, prompting her to file against him on February 14, 1995 a
complaint for ejectment and damages in the Municipal Trial Court in Cities of Cebu City (MTCC). The petitioner alleged that Amantes occupation of the
said land was only by the mere tolerance of Eliseo because the said property they inherited from their father had not yet been subdivided, and was thus
still co-owned by them; and that the respondents occupation had become illegal following his refusal to vacate despite repeated demands. MTCC ruled
in favour of petitioner. RTC reversed the decision of the MTCC. CA affirmed the RTC decision.

Issue: Whether there has been a valid action for unlawful detainer

Held:

A requisite for a valid cause of action of unlawful detainer is that the possession was originally lawful, but turned unlawful only upon the expiration of
the right to possess. To show that the possession was initially lawful, the basis of such lawful possession must then be established. With the averment
here that the respondents possession was by mere tolerance of the petitioner, the acts of tolerance must be proved, for bare allegation of tolerance did
not suffice. Considering that the allegation of the petitioners tolerance of the respondents possession of the disputed property was not established, the
possession could very well be deemed illegal from the beginning. In that case, her action for unlawful detainer has to fail. Even so, the Court would not
be justified to treat this ejectment suit as one for forcible entry because the complaint contained no allegation that this entry in the property had been
by force, intimidation, threats, strategy or stealth. The action for unlawful detainer is dismissed for being an improper remedy

CARMENCITA SUAREZ v. MR. AND MRS. FELIX E. EMBOY, JR. AND MARILOU P. EMBOYDELANTAR;
G.R. No. 187944 March 12, 2014
2
Facts:

A lot was issued in the name of Carmencita on February 9, 2005. The subject lot used to be a part of Lot No. 1907A,8 which was partitioned in the
following manner among the heirs of Spouses Carlos Padilla and Asuncion Pacres. A house, which is occupied by Respondents, stands in the subject lot.
The respondents claim that their mother, Claudia, had occupied the subject lot during her lifetime and it was earmarked to become her share in Lot No.
1907-A. They had thereafter stayed in the subject lot for decades after inheriting the same. In 2004, respondents were asked by their Vicente cousins to
vacate the subject lot and to transfer to Lot No. 1907A5, a landlocked portion sans a right of way. They refused to comply insisting that Claudias
inheritance pertained to Lot No. 1907A2. Not long after, the respondents received a demand letter from Petitioner requiring them to vacate the
subject lot. They were informed that Carmencita had already he subject lot from the formers relatives. However, the respondents did not heed the
demand. Instead, they examined the records pertaining to the subject lot and uncovered possible anomalies. Petitioner filed before MTCC and against
respondents a complaint for unlawful detainer. She alleged that she bought the subject lot from the registered owners who allowed the respondents to
occupy the same by mere tolerance. As their successorininterest, she claimed her entitlement to possession of the subject lot and the right to demand
from the respondents to vacate the same. the MTCC upheld petitioners claims. The respondents were ordered to vacate the subject lot and remove at
their expense all the improvements they had built thereon. The RTC affirmed MTCCs ruling.

The respondents argued that they have been occupying the subject lot in the concept of owners for several decades. Carmencita, on the other hand,
was a buyer in bad faith for having purchased the property despite the notice of lis pendens clearly annotated on the subject lots title. The respondents
insisted that the Heirs of Vicente, who had allegedly sold the subject lot to Carmencita, had never physically occupied the same. Hence, there was no
basis at all for Carmencitas claim that the respondents possession of the subject lot was by mere tolerance of the alleged owners. Respondents
presented a document where it showed that Vicente and his spouse, Dionesia, had waived their hereditary rights to Lot No. 1907A when he used the
same as a collateral in obtaining a loan. Prescinding from the above, the Heirs of Vicente no longer had ownership rights over the subject lot to convey
to Carmencita. The respondents also averred that Carmencitas complaint lacked a cause of action. The certification to file an action was issued in the
name of Carmencitas brother, who had no real rights or interests over the subject lot. Further, no demand to vacate was made upon the respondents.
The absence of such demand rendered the complaint fatally defective, as the date of its service should be the reckoning point of the oneyear period
within which the suit can be filed. CA reversed the decision and dismissed petitioners complaint for unlawful detainer.

Issues:

1. Whether or not Carmencitas complaint against the respondents had sufficiently alleged and proven a cause of action for unlawful detainer.
2. Whether or not the pendency of the respondents petition for nullification of partition of Lot No. 1907A and for the issuance of new
certificates of title can abate Carmencitas ejectment suit.

Ruling:

1. Carmencita had not amply alleged and proven that all the requisites for unlawful detainer are present in the case at bar. Without a doubt,
the registered owner of real property is entitled to its possession. However, the owner cannot simply wrest possession thereof from whoever
is in actual occupation of the property. To recover possession, he must resort to the proper judicial remedy and, once he chooses what action
to file, he is required to satisfy the conditions necessary for such action to prosper. In the case at bar, the first requisite that, initially,
possession of property by the defendant was by contract with or by tolerance of the plaintiff , is markedly absent. Carmencita failed to clearly
allege and prove how and when the respondents entered the subject lot and constructed a house upon it. Carmencita was likewise
conspicuously silent about the details on who specifically permitted the respondents to occupy the lot, and how and when such tolerance
came about. Instead, Carmencita cavalierly formulated a legal conclusion, sans factual substantiation, that (a) the respondents initial
occupation of the subject lot was lawful by virtue of tolerance by the registered owners, and (b) the respondents became deforciants
unlawfully withholding the subject lots possession after Carmencita had demanded for the former to vacate the property. When the complaint
fails to aver facts constitutive of forcible entry or unlawful detainer, as where it does not state how entry was effected or how and when
dispossession started, the remedy should either be an accion publiciana or accion reivindicatoria.
2. As an exception to the general rule, the respondents petition for nullification of the partition of Lot No. 1907A can abate Carmencitas suit
for unlawful detainer. Indisputably, the execution of the MCTC Decision would have resulted in the demolition of the house subject of the
ejectment suit; thus, by parity of reasoning, considerations of equity require the suspension of the ejectment proceedings. We should stress
that respondents claim to physical possession is based not on an expired or a violated contract of lease, but allegedly on mere tolerance.
Without in any way prejudging the proceedings for the quieting of title, we deem it judicious under the present exceptional circumstances to
suspend the ejectment case.

Paul Gabriel vs Crisologo


GR No. 204626 June 9, 2014

Facts:

Carmeling Crisologo (Crisologo), represented by her attorney-infact, Pedro Isican (Isican), filed her complaint for Recovery of Possession and/or
Ownership (Accion Publiciana) with Damages against Juliet B. Pulkera, Paul P. Gabriel, Ireneo C. Calwag, and Thomas L. Tingga-an (petitioners) before
the MTCC.

Crisologo alleged that she was the registered owner of the land and that sometime in 2006 she discovered that petitioners unlawfully entered, occupied
her properties. Upon the discovery of the illegal possession, her daughter and Isican personally went to the properties and verbally demanded that the
Gabriel vacate and remove their structures from the premises.

Petitioners countered that the titles of Crisologo were products of Civil Registration Case No. 1, Record 211, which were declared void by the Supreme
Court in Republic v. Marcos, and reiterated in Republic v. Marcos; that the said case was later enacted into law, Presidential Decree No. 1271.
Petitioners alleged that they had been in open, actual, exclusive, notorious, uninterrupted, and continuous possession of the subject land, in good faith.

Issue:

3
1.) Can the issue on ownership be raised in an action to recover possession (Accion Publiciana)?
2.) Who between the parties has a better right of possession over the subject lands?

Held:

1. The objective of the plaintiffs in accion publiciana is to recover possession only, not ownership. When parties, however, raise the issue of
ownership, the court may pass upon the issue to determine who between the parties has the right to possess the property. This adjudication,
nonetheless, is not a final and binding determination of the issue of ownership; it is only for the purpose of resolving the issue of possession,
where the issue of ownership is inseparably linked to the issue of possession. The adjudication of the issue of ownership, being provisional, is
not a bar to an action between the same parties involving title to the property. The adjudication, in short, is not conclusive on the issue of
ownership. Petitioners have raised the issue of ownership in their pleadings. They mainly argue that Crisologos titles on the subject properties
are void and that they have been in open, actual, exclusive, notorious, uninterrupted and continuous possession over the subject properties in
good faith.

The nullity of the decrees of registration and certificates of titles in Section 1 of P.D.
No. 1271 is not absolute. It has a provision stating "all certificates of titles issued on or before July 31, 1973 shall be considered valid and the
lands covered by them shall be deemed to have been conveyed in fee simple to the registered owners" upon
1) showing proof that the land covered by the subject title is not within any government, public or quasi-public reservation, forest, military or
otherwise, as certified by appropriating government agencies; and
2) compliance by the titleholder with the payment to the Republic of the Philippines of the correct assessed value of the land within the
required period.

In the case at bench, the records show that the subject parcels of land were registered on August 24, 1967. The titles are, thus, considered
valid although subject to the conditions set. But whether or not Crisologo complied with the said conditions would not matter because, this
would be a collateral attack on her registered titles. NOTE: Torrens Certificate of Title is IMMUNE from collateral attack.

2. Court holds that Crisologo has a better right of possession over the subject parcels of land. It cannot be denied that she bought the subject
properties from the previous owner in 1967, which was why the transfer certificates of title were subsequently issued in her name. Records
further show that she has been paying the realty taxes on the said properties since 1969. She likewise appointed Isican as administrator of
the disputed lands. More importantly, there is no question that she offered to sell to petitioners the portions of the subject properties occupied
by them. Hence, she deserves to be respected and restored to her lawful possession as provided in Article 539 of the New Civil Code.

Ang-CITY OF QUEZON CITY v. ERICTA


122 SCRA 759

FACTS: This is a petition for review seeking the reversal of the decision of the CFI of Rizal declaring Section 9 of Ordinance No. 6118 of the Quezon City
Council null and void. Said section in the provision mandates that at least six (6) percent of the total area of the memorial park cemetery shall be set
aside for charity burial of deceased persons who are paupers and have been residents of Quezon City for at least 5 years prior to their death. Petitioners
argue that the taking of the respondents property is a valid and reasonable exercise of police power.

ISSUE: Is Section 9 of the ordinance in question a valid exercise of the police power?

HELD:
No. There is no reasonable relation between the setting aside of at least six percent of the total area of private cemeteries for charity burial grounds for
deceased paupers and the promotion of health, morals, good order, safety, or the general welfare of the people.

It is a well-settled principle, growing out of the nature of well-ordered and society, that every holder of property, however absolute and may be his title,
holds it under the implied liability that his use of it shall not be injurious to the equal enjoyment of others having an equal right to the enjoyment of
their property, nor injurious to the rights of the community. A property in the state is held subject to its general regulations, which are necessary to the
common good and general welfare. Rights of property, like all other social and conventional rights, are subject to such reasonable limitations in their
enjoyment as shall prevent them from being injurious, and to such reasonable restraints and regulations, established by law, as the legislature, under
the governing and controlling power vested in them by the constitution, may think necessary and expedient. The state, under the police power, is
possessed with plenary power to deal with all matters relating to the general health, morals, and safety of the people, so long as it does not contravene
any positive inhibition of the organic law and providing that such power is not exercised in such a manner as to justify the interference of the courts to
prevent positive wrong and oppression.

The ordinance is actually taking without compensation of a certain area from a private cemetery to benefit paupers who are charges of the municipal
corporation. Instead of building or maintaining a public cemetery for these purposes, the city passes the burden to private cemeteries. This is
expropriation, which requires payment of just compensation.

THE OFFICE OF THE SOLICITOR GENERAL vs. AYALA LAND INCORPORATED, ROBINSON'S LAND CORPORATION, SHANGRILA PLAZA
CORPORATION and SM PRIME HOLDINGS, INC.,
G.R. No. 177056 September 18, 2009

Facts: The shopping malls operated or leased out by respondents have parking facilities for all kinds of motor vehicles, either by way of parking spaces
inside the mall buildings or in separate buildings and/or adjacent lots that are solely devoted for use as parking spaces. Respondents expend for the
maintenance and administration of their respective parking facilities. The Senate Committees find that the collection of parking fees by shopping malls is
contrary to the National Building Code and is therefor illegal.

4
The Committees on Trade and Commerce and Justice and Human Rights hereby recommend that Congress should amend and update the National
Building Code to expressly prohibit shopping malls from collecting parking fees by at the same time, prohibit them from invoking the waiver of liability.
Threatened with such information, the respondent filed a petition for declaratory relief.

The RTC rendered a decision and stated that, to compel respondent malls to provide parking spaces for free can be considered as an unlawful taking of
property right without just compensation. Parking spaces in shopping malls are privately owned and for their use, the mall operators collect fees. The
legal relationship could be either lease or deposit. The mall owners have the right to collect money which translates into income. Should parking spaces
be made free, this right of mall owners shall be gone. The presence of parking spaces can be looked at as necessary facilities to entice the public to
increase patronage of their malls because without parking spaces, going to their malls will be inconvenient. These are, however, business considerations
which mall operators will have to decide for themselves. They are not sufficient to justify a legal conclusion, as the OSG would like the Court to adopt
that it is the obligation of the mall owners to provide parking spaces for free. The OSG appealed the case to the CA and the CA affirmed RTCs decision.

Issues: WON respondents are mandated to provide free parking by Section 803 of the National Building Code and Rule XIX of the IRR.

Ruling:

The explicit directive of the afore-quoted statutory and regulatory provisions is that respondents should provide parking and loading spaces, in
accordance with the minimum ratio of one slot per 100 square meters of shopping floor area. There is nothing therein pertaining to the collection
(or non-collection) of parking fees by respondents. In fact, the term "parking fees" cannot even be found at all in the entire National Building
Code and its IRR. Since Section 803 of the NBC and Rule XIX of its IRR do not mention parking fees, then simply, said provisions do not regulate the
collection of the same.

Also, The Court cannot simply assume, as the OSG has apparently done, that the traffic congestion in areas around the malls is due to the fact that
respondents charge for their parking facilities, thus, forcing vehicle owners to just park in the streets. The Court notes that despite the fees charged by
respondents, vehicle owners still use the mall parking facilities, which are even fully occupied on some days. Vehicle owners may be parking in the
streets only because there are not enough parking spaces in the malls, and not because they are deterred by the parking fees charged by respondents.
Free parking spaces at the malls may even have the opposite effect from what the OSG envisioned: more people may be encouraged by the free
parking to bring their own vehicles, instead of taking public transport, to the malls; as a result, the parking facilities would become full sooner, leaving
more vehicles without parking spaces in the malls and parked in the streets instead, causing even more traffic congestion.

The Court finds, however, that in totally prohibiting respondents from collecting parking fees from the public for the use of the mall parking facilities, the
State would be acting beyond the bounds of police power. Although in the present case the prohibition against their collection of parking fees from the
public, for the use of said facilities, is tantamount to a taking or confiscation of their properties. The State is not only requiring that respondents devote
a portion of the latters properties for use as parking spaces, but is also mandating that they give the public access to said parking spaces for free. Such
is already an excessive intrusion into the property rights of respondents. Not only are they being deprived of the right to use a portion of their properties
as they wish, they are further prohibited from profiting from its use or even just recovering therefrom the expenses for the maintenance and operation
of the required parking facilities.

City of Ozamis vs Lumapas


GR No. 30727
July 15, 1975

Facts:
Respondent Serapio S. Lumapas is an operator of transportation buses for passengers and cargoes, under the name of Romar Line, with Ozamiz City
and Pagadian, Zamboanga del Sur, as terminal points, by virtue of a certificate of public convenience issued to him by the Public Service Commission.
The Municipal Board of Ozamiz City enacted Ordinance 446 AN ORDINANCE IMPOSING PARKING FEES FOR EVERY MOTOR VEHICLE PARKED ON ANY
PORTION OF THE EXISTING PARKING SPACE IN THE CITY OF OZAMIZ.
After approval of the above-quoted ordinance, the City of Ozamiz began collecting the prescribed parking' fees and collected from respondent-appellee
Serapio S. Lumapas, who had paid under protest, the parking fees at One Peso (P1.00) for each of his buses, from October 1964 to January 1967, or an
aggregate amount of P1,259.00 for which official receipts were issued by petitioner.
About four (4) years later, or on January 11, 1968, respondent Serapio S. Lumapas filed a complaint, dated August 3, 1967 3 against the City of Ozamiz
for recovery of parking fees, alleging, among others, that said Ordinance No. 466 is ulta vires, and praying that judgment be issued (1) nullifying
Ordinance No. 466, series of 1964, and (2) ordering the Municipal Board to appropriate the amount of P1,459.00 for the reimbursement of P1,259.00 he
had paid as parking fees, plus P200.00 as attorney's fees.

Issue:
Whether the imposition of parking fees in the existing parking space in the City of Ozamiz is a valid exercise of police power

Held:
Under Sec. 15[Y] of the Ozamiz City Charter (Rep. Act No. 321), the municipal board has the power "... to regulate the use of streets, avenues, alleys,
sidewalks, wharves, piers, parks, cemeteries and other public places; ...", and in subsection [nn] of the same section 15, the authority "To enact all
ordinances it may deem necessary and proper for the sanitation and safety, the furtherance of prosperity and the promotion of the morality, peace,
good order, comfort, convenience, and general welfare of the city and its inhabitants, and such others as may be necessary to carry into effect and
discharge the powers and duties conferred by this Charter ..." By this express legislative grant of authority, police power is delegated to the municipal
corporation to be exercised as a governmental function for municipal purposes.
As adverted to above, the Municipal Board of Ozamiz City is expressly granted by its Charter the power to regulate the use of its streets. The ordinance
in question appears to have been enacted in pursuance of this grant.

ATO and MCIAA vs. GOPUCO


G.R. No. 158563
June 30, 2005
5
FACTS:

Respondent Apolonio Gopuco, Jr. was the owner of Cadastral Lot No. 72 located in the vicinity of the Lahug Airport in Cebu City. His lot, together with
the other lots in the area, was expropriated for the purpose of expanding the Lahug Airport. Subsequently, when the Mactan International Airport
commenced operations, the Lahug Airport was ordered closed and Lot No. 72 was thus virtually abandoned.

On 06 August 1992, Apolonio Gopuco, Jr. filed an amended complaint for recovery of ownership of Lot No. 72. He maintained that by virtue of the
closure of the Lahug Airport, the original purpose for which the property was expropriated had ceased or otherwise been abandoned, and title to the
property had therefore reverted to him. On 20 May 1994, the trial court rendered a Decision dismissing the complaint.

The trial court held:The fact of abandonment or closure of the Lahug Airport admitted by the defendant did not by itself result in the reversion of the
subject property back to the plaintiff. Nor did it vest in the plaintiff the right to demand reconveyance of said property. Furthermore, the abandonment
or non-use of the real property, does not ipso facto give to the previous owner of said property any right to recover the same.
It further asserts that said judgment vested absolute and unconditional title in the government, specifically on the petitioners, there having been no
condition whatsoever that the property should revert to its owners in case the Lahug Airport should be abandoned. Aggrieved by the holding of the trial
court, Gopuco appealed to the Court of Appeals, which overturned the RTC decision holding that: While, in the expropriation case, there is no explicit
statement that the land is expropriated with the condition that when the purpose is ended the property shall return to its owner, the full import of the
decision suggests that the expropriation was granted because there is no clear showing that Lahug Airport will be closed, the moment Mactan
International Airport is put to operation. It stands to reason that should that public use be abandoned, then the expropriated property should revert
back to its former owner.

Moreover, the foundation of the right to exercise the power of eminent domain is genuine necessity. Condemnation is justified only if it is for the public
good and there is genuine necessity of a public character. Thus, when such genuine necessity no longer exists as when the State abandons the property
expropriated, government interest must yield to the private right of the former land owner, whose property right was disturbed as a consequence of the
exercise of eminent domain. Justice, equity and fair play demand that the property should revert back to plaintiff-appellant upon paying the reasonable
value of the land to be based on the prevailing market value at the time of judicial demand to recover the property. If the State expects landowners to
cooperate in its bid to take private property for its public use, so must it apply also the same standard, to allow the landowner to reclaim the property,
now that the public use has been abandoned. Petitioners filed for a Motion for Reconsideration but were denied, hence this petition.

ISSUE: When private land is expropriated for a particular public use, and that particular public use is abandoned, does the land so expropriated return
to its former owner?

RULING:

The answer to that question depends upon the character of the title acquired by the expropriator.
If, for example, land is expropriated for a particular purpose, with the condition that when that purpose is ended or abandoned the property shall return
to its former owner, then, of course, when the purpose is terminated or abandoned the former owner reacquires the property so expropriated. If upon
the contrary, however, the decree of expropriation gives to the entity a fee simple title, then of course, the land becomes the absolute property of the
expropriator. When land has been acquired for public use in fee simple, unconditionally, either by the exercise of eminent domain or by purchase, the
former owner retains no rights in the land, and the public use may be abandoned or the land may be devoted to a different use, without any
impairment of the estate or title acquired, or any reversion to the former owner

There was a compromise agreement before the final judgment of the expropriation proceeding. However, Gopuco was not a party thereto and so he
cannot legally invoke the same. Heirs Moreno v. MCIAA was also mentioned. Petitioners won because, In this case, Supreme Court held that the
predicament of petitioners therein involved a constructive trust "akin to the implied trust referred to in Art. 1454 of the Civil Code." And that there is a
preponderant proof as found by the trial court of the existence of the right of repurchase in favor of petitioners. Chiongbian case was also mentioned.
Chiongbian put forth inadmissible and inconclusive evidence, and neither has Gopuco, in the present case, adduced any evidence at all concerning a
right of repurchase in his favor.

HEIRS OF MORENO vs. MACTAN - CEBU INTERNATIONAL AIRPORT AUTHORITY


G.R. No. 156273
October 15, 2003

Facts:
Petitioners herein, are the successors-in-interest of the former registered owners of 2 parcels of land situated in Lahug, Cebu City. In 1949 the National
Airport Corporation as the predecessor agency of respondent MCIAA wanted to acquire the said lots. To entice the landowners to cede their properties,
the government assured them that they could repurchase their lands once Lahug Airport was closed or its operations transferred to Mactan Airport.
Some of the landowners executed deeds of sale with right of repurchase in favor of the government but many others, including the Morenos refused the
offer because the payment was perceived to be way below the market price. Later, the Civil Aeronautics Administration as the successor agency of the
National Airport Corporation filed a complaint for the expropriation of the lots. The Trial Court condemned the lots for public use upon payment of just
compensation. Petitioners predecessors were paid P7,065.00 with consequential damages by way of legal interest from 16 November 1947. No appeal
was taken from the Decision and the judgment of condemnation became final and executory. Thereafter, the certificates of title for these parcels of land
were issued in the name of the Republic of the Philippines, which under RA 6958 (1990) were subsequently transferred in favor of respondent MCIAA.
At the end of 1991, Lahug Airport ceased operations as the Mactan Airport was opened for incoming and outgoing flights. The lots which had been
expropriated for the extension of Lahug Airport were not utilized. In fact, no expansion of Lahug Airport was undertaken by MCIAA and its
predecessors-in-interest. Hence, petitioners wrote then President Fidel V. Ramos and the airport manager begging them for the exercise of their alleged
right to repurchase the lots. Their pleas were not heeded so petitioners filed a complaint for reconveyance and damages with RTC against MCIAA to
compel the repurchase the lots. In the main, petitioners averred that they had been convinced by the officers of the predecessor agency of MCIAA not
to oppose the expropriation proceedings since in the future they could repurchase the properties if the airport expansion would not push through.
MCIAA did not object to petitioners evidence establishing these allegations.

6
When the civil case was pending, one Richard E. Enchuan alleged that he acquired through deeds of assignment the rights of some of herein petitioners
over the lots. The DPWH also sought to intervene in the civil case claiming that it leased in good faith one of the lots from the predecessor agencies of
respondent MCIAA and that it built thereon its Regional Equipment Services and its Region 7 Office.

The RTC granted the petitioners the right to repurchase the properties at the amount pegged as just compensation but subject to the alleged property
rights of Enchuan and the leasehold of DPWH. The trial court opined that the expropriation became illegal or functus officio when the purpose for which
it was intended was no longer there. Respondent appealed to CA. The CA reversed the RTCs decision on the ground that the judgment of
condemnation was unconditional so that the rights gained therefrom by respondent MCIAA were indicative of ownership in fee simple.

Issue: WON the petitioners have the right to reconvey the said lots?

Ruling:

Yes. Petitioners allege that their right to equal protection of the laws would be infringed if some landowners are given the right to repurchase their
former properties even as they are denied the exercise of such prerogative. On the other hand, MCIAA alleged that "there is only one instance when
expropriated land may be repurchased by its previous owners, and that is, if the decision of expropriation itself provides [the] condition for such
repurchase."

In resolving this dispute, we must reckon with the rulings of this Court in Fery v. Municpality of Cabanatuan and MCIAA v. Court of Appeals, which
define the rights and obligations of landowners whose properties were expropriated when the public purpose for which eminent domain was exercised
no longer subsists. In Fery, we declared that the government acquires only such rights in expropriated parcels of land as may be allowed by the
character of its title over the properties - If the land is expropriated for a particular purpose, with the condition that when that purpose is ended or
abandoned the property shall return to its former owner, then, of course, when the purpose is terminated or abandoned the former owner reacquires
the property so expropriated.

No doubt, the return or repurchase of the condemned properties of petitioners could be readily justified as the manifest legal effect or consequence of
the trial courts underlying presumption that "Lahug Airport will continue to be in operation" when it granted the complaint for eminent domain and the
airport discontinued its activities. The predicament of petitioners involves a constructive trust, one that is akin to the implied trust referred to in Art.
1454 of the Civil Code, "If an absolute conveyance of property is made in order to secure the performance of an obligation of the grantor toward the
grantee, a trust by virtue of law is established. If the fulfillment of the obligation is offered by the grantor when it becomes due, he may demand the
reconveyance of the property to him."

In the case at bar, petitioners conveyed the lots to the government with the latter obliging itself to use the realties for the expansion of Lahug Airport;
failing to keep its bargain, the government can be compelled by petitioners to reconvey the parcels of land to them, otherwise, petitioners would be
denied the use of their properties upon a state of affairs that was not conceived nor contemplated when the expropriation was authorized.

The rights and obligations between the constructive trustee and the beneficiary, in this case, respondent MCIAA and petitioners over the lots, are
echoed in Art. 1190 of the Civil Code, "When the conditions have for their purpose the extinguishment of an obligation to give, the parties, upon the
fulfillment of said conditions, shall return to each other what they have received" Hence, MCIAA as representative of the State is obliged to reconvey the
lots to petitioners who shall hold the same subject to existing liens thereon, i.e., leasehold right of DPWH. In return, petitioners as if they were plaintiff
beneficiaries of a constructive trust must restore to MCIAA what they received as just compensation for the expropriation of the lots, i.e.,P7,065.0 with
consequential damages by way of legal interest. Petitioners must likewise pay MCIAA the necessary expenses it may have incurred in sustaining the
properties and the monetary value of its services in managing them to the extent that petitioners will be benefited thereby.

The government however may keep whatever income or fruits it may have obtained from the parcels of land, in the same way that petitioners need not
account for the interests that the amounts they received as just compensation may have earned in the meantime. If petitioners do not want to
appropriate such improvements or respondent does not choose to sell them, the improvements would have to be removed without any obligation on the
part of petitioners to pay any compensation to respondent MCIAA for whatever it may have tangibly introduced therein.

Lunod et al vs Meneses
GR No. 4223
August 19, 1908

Facts:
Lunod et al were owners of land situated in Maytunas and Balot , Bulacan. These lands are situated near a small adjoining lake named Calalaran which
are located in places relatively higher than Sitio Paraaanan where the land and fishpond of the Meneses are situated. The land of Meneses was near the
Taliptip River. During rainy seasons the water which falls on the lands of Lunod flows to the Calalaran Lake. The outlet of this water is only through
Paraanan. Since time immemorial there existed a favour of the rice fields of the plaintiff a statutory easement permitting the flow of water over the said
land in Paraanan into the Taliptip River. Sometime in 1901, Menesis created a dam along the boundary of his fishpond in Paraanan, thereby impeding
the outlet of the waters that fold the fields at Calalaran to the serious detriment of the growing crops. Menesis alleged that there was no statutory
easement that existed.

Issues:
1. Was there a statutory easement?
2. Can Menesis, in the exercise of his ownership, construct works necessary to make and maintain a fishpond within his own land to the
detriment of other people?

Held:

Statutory easement was not constituted by agreement between the parties. It is of statutory nature, and the law has imposed it for the common public
utility in view of the difference in the altitude of the lands in Barrio Bambang.

7
The Civil Code provides: ART552: Lower estates must receive the waters which naturally and without the intervention of man descend from the higher
estates, as well as the stone or earth which they carry with them. Neither may the owner of the lower estate construct works preventing this
easement, nor the one of the higher estate works increasing the burden

The Law of Waters provides: ART 111 Lands situated at a lower level are subject to receive the waters that flow naturally, without the work of man,
from the higher lands together with the stone or earth which they carry with them

According to the provisions of law above referred to, Menesis, had no right to construct the works, nor the dam which blocks the passage, through his
lands and the outlet to the Taliptip River, of the waters which flood the higher lands of the plaintiffs; and having done so, to the detriment of the
easement charged on his estate, he has violated the law which protects and guarantees the respective rights and regulated the duties of the owners of
the fields in Calalaran and Paraanan.
VALISNO vs. ADRIANO
G.R. No. L-37409
May 23, 1988

FACTS:

Valisno bought a parcel of land from Honorata Adriano Francisco, the sister of herein appellee. At the time of the sale, the land was irrigated by water
from the Pampanga River through a canal traversing the appellee's land. On December 16, 1959, the appellee levelled a portion of the irrigation canal
so that the appellant was deprived of the irrigation water and prevented from cultivating his land. The appellant filed in the Bureau of Public Works and
Communications a complaint for deprivation of water rights and on June 20, 1960, appellant also filed a complaint for damages in the Court of First
Instance. Subsequently, a decision was rendered by the Bureau ordering Adriano to reconstruct the irrigation canal but instead of restoring the
irrigation canal, the appellee asked for a reinvestigation of the case. A reinvestigation was granted and the decision was reversed. Adriano contention
was that neither his late father nor his sister Honorata possessed water rights for the land which she sold to the appellant; that he (the appellee)
applied for water rights for his land on his own; and that he had a perfect right to level his land for his own use because he merely allowed his sister to
use his water rights when she still owned the adjacent land.

The Secretary held that non-use of the water right by the appellees father for a period of more than five years extinguished the grant by operation of
law; hence the water rights did not form part of his hereditary estate which his heirs partitioned among themselves. Valisno, as vendee of the land
which Honorata received from her father's estate, did not acquire any water rights with the land purchased. The trial court also held that the plaintiff
had no right to pass through the defendant's land to draw water from the Pampanga River. It pointed out that under Section 4 of the Irrigation Law,
controversies between persons claiming right to water from a stream are within the jurisdiction of the Secretary of Public Works and his decision on the
matter is final, unless an appeal is taken to the proper court within thirty days. The plaintiff's motion for reconsideration of the decision was denied by
the trial court. The plaintiff appealed to the Court of Appeals which certified the case to Supreme Court.

The plaintiff-appellant argues that while the trial court correctly held that the Secretary of Public Works may legally decide who between the parties is
entitled to apply for water rights under the Irrigation Act, it erred in ruling that the Secretary has authority to hear and decide the plaintiff 's claim for
damages for the defendant's violation of his (plaintiff's) right to continue to enjoy the easement of aqueduct or water through the defendant's land
under Articles 642, 643, and 646 of the Civil Code.

ISSUE: Does the plaintiff have water right specifically to the water canal?

RULING

Yes. Water rights, such as the right to use a drainage ditch for irrigation purposes, which are appurtenant to a parcel of land, pass with the conveyance
of the land, although not specifically mentioned in the conveyance. The purchaser's easement of necessity in a water ditch running across the grantor's
land cannot be defeated even if the water is supplied by a third person. As an easement of waters in favor of the appellant has been established, he is
entitled to enjoy it free from obstruction, disturbance or wrongful interference such as the appellee's act of levelling the irrigation canal to deprive him
of the use of water from the Pampanga River.

THE ROMAN CATHOLIC ARCHBISHOP OF MANILA vs. HON. COURT OF APPEALS, THE ESTATE OF DECEASED SPOUSES EUSEBIO DE
CASTRO and MARTINA RIETA
G.R. No. 77450
June 19, 1991

Facts: Private respondents filed a complaint for nullification of deed of donation, rescission of contract and reconveyance of real property with damages
against petitioners and alleged that on August 23, 1930, the spouses Eusebio de Castro and Martina Rieta, executed a deed of donation in favor of
Roman Catholic Archbishop of Manila covering a parcel of land. The deed of donation allegedly provides that the donee shall not dispose or sell the
property within a period of 100 years from the execution of the deed of donation, otherwise a violation of such condition would render ipso facto null
and void the deed of donation and the property would revert to the estate of the donors. It is further alleged that on or about June 30, 1980, and while
still within the prohibitive period to dispose of the property, petitioner Roman Catholic Bishop of Imus, in whose administration all properties within the
province of Cavite owned by the Archdiocese of Manila was allegedly transferred in favor of petitioners in consideration of the sum of P114,000.00. As a
consequence of the sale, a OCT was issued in the name of said petitioner spouses.

The trial court issued an order dismissing the complaint on the ground that the cause of action has prescribed. Respondents thereafter appealed to the
Court of Appeals. CA held that action has not yet prescribed and REINSTATED and REMANDED to the lower court for further proceedings.

Issues:
1. Whether or not the action for rescission of contracts (deed of donation and deed of sale) has prescribed
2. Whether or not the dismissal of the action for rescission of contracts on the ground of prescription carries with it the dismissal of the main
action for reconveyance of real property.

8
Ruling:

Although it is true that under Article 764 of the Civil Code an action for the revocation of a donation must be brought within 4 years from the non-
compliance of the conditions of the donation, the same is not applicable in the case at bar. The deed of donation involved herein expressly provides for
automatic reversion of the property donated in case of violation of the condition therein, hence a judicial declaration revoking the same is not necessary,
As aptly stated by the Court of Appeals: By the very express provision in the deed of donation itself that the violation of the condition thereof would
render ipso facto null and void the deed of donation, WE are of the opinion that there would be no legal necessity anymore to have the donation
judicially declared null and void for the reason that the very deed of donation itself declares it so. For where it otherwise and that the donors and the
donee contemplated a court action during the execution of the deed of donation to have the donation judicially rescinded or declared null and void
should the condition be violated, then the phrase reading "would render ipso facto null and void" would not appear in the deed of donation.

In support of its aforesaid position, respondent court relied on the rule that a judicial action for rescission of a contract is not necessary where the
contract provides that it may be revoked and cancelled for violation of any of its terms and conditions. It called attention to the holding that there is
nothing in the law that prohibits the parties from entering into an agreement that a violation of the terms of the contract would cause its cancellation
even without court intervention. Article 732 of the Civil Code provides that donations inter vivos shall be governed by the general provisions on contracts
and obligations in all that is not determined in Title III, Book III on donations. Now, said Title III does not have an explicit provision on the matter of a
donation with a resolutory condition and which is subject to an express provision that the same shall be considered ipso facto revoked upon the breach
of said resolutory condition imposed in the deed therefor, as is the case of the deed presently in question. The suppletory application of the foregoing
doctrinal rulings to the present controversy is consequently justified.

The rationale for the foregoing is that in contracts providing for automatic revocation, judicial intervention is necessary not for purposes of obtaining a
judicial declaration rescinding a contract already deemed rescinded by virtue of an agreement providing for rescission even without judicial intervention,
but in order to determine whether or not the rescission was proper. When a deed of donation expressly provides for automatic revocation and reversion
of the property donated, the rules on contract and the general rules on prescription should apply. Since Article 1306 of said Code authorizes the parties
to a contract to establish such stipulations not contrary to law, morals, good customs, public order or public policy, we are of the opinion that, at the
very least, that stipulation of the parties providing for automatic revocation of the deed of donation, without prior judicial action for that purpose, is
valid subject to the determination of the propriety of the rescission sought.

On the foregoing ratiocinations, the Court of Appeals committed no error in holding that the cause of action of herein private respondents has not yet
prescribed since an action to enforce a written contract prescribes in 10 years. We find that although the action filed by private respondents may not be
dismissed by reason of prescription, the same should be dismissed on the ground that private respondents have no cause of action against petitioners.
The cause of action of private respondents is based on the alleged breach by petitioners of the resolutory condition in the deed of donation that the
property donated should not be sold within a period of 100 years from the date of execution of the deed of donation. Said condition, in our opinion,
constitutes an undue restriction on the rights arising from ownership of petitioners and is, therefore, contrary to public policy. Donation, as a mode of
acquiring ownership, results in an effective transfer of title over the property from the donor to the donee. Once a donation is accepted, the donee
becomes the absolute owner of the property donated. Although the donor may impose certain conditions in the deed of donation, the same must not be
contrary to law, morals, good customs, public order and public policy. The condition imposed in the deed of donation in the case before us constitutes
a patently unreasonable and undue restriction on the right of the donee to dispose of the property donated, which right is an
indispensable attribute of ownership. Such a prohibition against alienation, in order to be valid, must not be perpetual or for an unreasonable
period of time.

Certain provisions of the Civil Code illustrative of the aforesaid policy may be considered applicable by analogy. Under the third paragraph of Article 494,
a donor or testator may prohibit partition for a period which shall not exceed 20 years. Article 870, on its part, declares that the dispositions of the
testator declaring all or part of the estate inalienable for more than twenty (20) years are void.
We hold that the prohibition in the deed of donation against the alienation of the property for an entire century, being an unreasonable emasculation
and denial of an integral attribute of ownership, should be declared as an illegal or impossible condition within the contemplation of Article 727 of the
Civil Code. Consequently, as specifically stated in said statutory provision, such condition shall be considered as not imposed. No reliance may
accordingly be placed on said prohibitory paragraph in the deed of donation. The net result is that, absent said proscription, the deed of sale supposedly
constitutive of the cause of action for the nullification of the deed of donation is not in truth violative of the latter hence, for lack of cause of action, the
case for private respondents must fail. It will readily be noted that the provision in the deed of donation against alienation of the land for one hundred
(100) years was the very basis for the action to nullify the deed of donation.

German Management & Services, Inc. Vs Hon. CA


GR No. 76216-17
SEPTEMBER 14, 1989

FACTS:
Spouses Cynthia Cuyegkeng Jose and Manuel Rene Jose, residents of Pennsylvania, Philadelphia, USA are the owners of a parcel of land situated in Sitio
Inarawan, San Isidro, Antipolo, Rizal, with an area of 232,942 square meters.
On February 26, 1982, the spouses Jose executed a special power of attorney authorizing petitioner German Management Services to develop their
property covered by TCT No. 50023 into a residential subdivision.
Finding that part of the property was occupied by private respondents and twenty other persons, petitioner advised the occupants to vacate the
premises but the latter refused.
1.) Petitioner forcibly removed and destroyed the barbed wire fence enclosing the farmholdings of the private respondents WITHOUT NOTICE
2.) and bulldozed the rice, corn, fruit bearing trees and other crops of private respondents by means of force, violence and intimidation in
violation of PD 1038
3.) TRESPASSING, COERCING AND THREATENING TO HARASS, REMOVE AND EJECT PRIVATE RESPONDENTS FROM THEIR RESPECTIVE
FARMHOLDINGS IN VIOLATION OF PD NO. 316, 583,815 AND 1028.

Private respondents' claimed that they were in peaceable possession of the land. Such was manifested by the fact that they even planted rice, corn and
fruit bearing trees twelve to fifteen years.

Issue:
9
Is the principle of self-help applicable in cases when possession of a property has already been lost?

Held:
In the case at bar, it is undisputed that at the time the petitioner enetered the property, private respondents were already in possession thereof. There
is no evidence that the souses Jose were ever in the possession of the subject property. On the contrary, private respondents peaceable possession was
manifested by the fact that they even planted rice, corn, fruit and bearing trees twelve to fifteen years prior to petitioners act of destroying their crops.
Both the Municipal Trial Court and the Regional Trial Court have rationalized petitioners drastic action of bulldozing and destroying the crops of private
respondents on the bases of the doctrine of self help enunciated in article 429 of the NCC. SUCH JUSTIFICATION IS UNAVAILING because the doctrine
of self help can only be exercised at the time of actual or threatened dispossession which is absent in the case at bar. When the possession has already
lost the owner must resort to judicial process for the recovery of property. This is clear from Article 536 of the Civil Code which states, "(I)n no case
may possession be acquired through force or intimidation as long as there is a possessor who objects thereto. He who believes that he has an action or
right to deprive another of the holding of a thing, must invoke the aid of the competent court, if the holder should refuse to deliver the thing.

NPC v. LUCMAN IBRAHIM


GR No. 149295
September 23, 2003

FACTS: Respondent Lucman, in his personal capacity and in behalf of his co-heirs, instituted an action against NAPOCOR for recovery of possession of
land and damages before the RTC of Lanao del Sur.
In their complaint, Ibrahim and his co-heirs claimed that they were owners of several parcels of land and that sometime in 1978, NAPOCOR,
through alleged stealth and without respondents knowledge and prior consent, took possession of the sub-terrain area of their lands and constructed
therein underground tunnels. The existence of the tunnels was only discovered sometime in July 1992 by respondents and then later confirmed in
November 1992 by NAPOCOR.
Respondents demanded that NAPOCOR pay damages and vacate the sub-terrain portion of their lands but the latter refused to vacate much
less pay damages. Respondents further averred that the construction of the underground tunnels has endangered their lives and properties
as Marawi City lies in an area of local volcanic and tectonic activity.
Disputing respondents claim, NAPOCOR filed an answer with counterclaim denying the material allegations of the complaint and interposing
affirmative and special defenses, namely that (1) there is a failure to state a cause of action since respondents seek possession of the sub-terrain
portion when they were never in possession of the same, (2) respondents have no cause of action because they failed to show proof that they were the
owners of the property, and (3) the tunnels are a government project for the benefit of all and all private lands are subject to such easement as may be
necessary for the same.
This case revolves around the propriety of paying just compensation to respondents. The threshold issue of whether respondents are entitled
to just compensation hinges upon who owns the sub-terrain area occupied by petitioner. Petitioner maintains that the sub-terrain portion where the
underground tunnels were constructed does not belong to respondents because, even conceding the fact that respondents owned the property, their
right to the subsoil of the same does not extend beyond what is necessary to enable them to obtain all the utility and convenience that such property
can normally give. Petitioner asserts that respondents were still able to use the subject property even with the existence of the tunnels. Petitioner
concludes that the underground tunnels 115 meters below respondents property could not have caused damage or prejudice to respondents and their
claim to this effect was, therefore, purely conjectural and speculative.

ISSUE: Who owns the sub-terrain portion of the property?

HELD:

The sub-terrain portion of the property belongs to the respondents. This conclusion is drawn from Article 437 of the Civil Code which provides:

ART. 437. The owner of a parcel of land is the owner of its surface and of everything under it, and he can construct thereon any works or make
any plantations and excavations which he may deem proper, without detriment to servitudes and subject to special laws and ordinances. He cannot
complain of the reasonable requirements of aerial navigation.

Thus, the ownership of land extends to the surface as well as to the subsoil under it. The Court has held that if the government takes property without
expropriation and devotes the property to public use, after many years, the property owner may demand payment of just compensation in the event
restoration of possession is neither convenient nor feasible. This is in accordance with the principle that persons shall not be deprived of their property
except by competent authority and for public use and always upon payment of just compensation. Petitioner contends that the underground tunnels in
this case constitute an easement upon the property of respondents which does not involve any loss of title or possession. The manner in which the
easement was created by petitioner, however, violates the due process rights of respondents as it was without notice and indemnity to them and did
not go through proper expropriation proceedings. Petitioner could have, at any time, validly exercised the power of eminent domain to acquire the
easement over respondents property as this power encompasses not only the taking or appropriation of title to and possession of the expropriated
property but likewise covers even the imposition of a mere burden upon the owner of the condemned property. Landowners cannot be deprived of their
right over their land until expropriation proceedings are instituted in court. The court must then see to it that the taking is for public use, that there is
payment of just compensation and that there is due process of law.

The acquisition of the easement is not without expense. The underground tunnels impose limitations on respondents use of the property for an
indefinite period and deprive them of its ordinary use. Respondents are clearly entitled to the payment of just compensation. Notwithstanding the fact
that petitioner only occupies the sub-terrain portion, it is liable to pay not merely an easement fee but rather the full compensation for land. This is so
because in this case, the nature of the easement practically deprives the owners of its normal beneficial use. Respondents, as the owners of the
property thus expropriated, are entitled to a just compensation which should be neither more nor less than the money equivalent of said property.

REPUBLIC vs. DE LA ROSA


G.R. No. L-43938
April 15, 1988

FACTS:

10
This is a consolidated case involving a parcel of land. When De La Rosa applied for registration of said land, his application was opposed by 1.) Benguet
Consolidated, Inc., 2.) Atok Big Wedge Corporation, and 3.) The Bureau of Forestry and Development. De la Rosa and family bought the said property
from Balbalio and Alberto who claimed ownership right based on prescription since they were in possession of the said parcel of land sometime after the
Liberation. Benguet Consolidated, Inc. claimed its ownership rights based on the June Bug mineral claim it bought sometime in 1934. Said claim was
perfected sometime in 1909. Atok Big Wedge Corporation claimed its ownership right based on Emma and Fredie mineral claims which it bought
sometime in 1931. Said claim was perfected around January of 1931. These three parties were all claiming to be in possession of the subject land. De la
Rosa and family were cultivating its surface while Benguet and Atok were doing some mining on the sub-surface. The Bureau of Forestry and
Development argued that the land sought to be registered was covered by the Central Cordillera Forest Reserve under Proclamation No. 217
datedFebruary 16, 1929. Moreover, by reason of its nature, it was not subject to alienation under the Constitutions of 1935 and 1973.

ISSUE: Who had a better right, the farmer, the miners, or the state?

RULING

With respect to BFDs claim, it is true that the subject property was considered forest land and included in the Central Cordillera Forest Reserve, but this
did not impair the rights already vested in Benguet and Atok at that time. The June Bug mineral claim of Benguet and the Fredia and Emma
mineralclaims of Atok having been perfected prior to the approval of the Constitution of the Philippines of 1935. They were removed from the public
domain and had become private properties of Benguet and Atok.

With respect to De la Rosas claim, under the theory of the respondent court, the surface owner will be planting on the land while the mining locator will
be boring tunnels underneath. The farmer cannot dig a well because he may interfere with the operations below and the miner cannot blast a tunnel
lest he destroy the crops above. How deep can the farmer, and how high can the miner, go without encroaching on each other's rights? Where is the
dividing line between the surface and the sub-surface rights?

The Court feels that the rights over the land are indivisible and that the land itself cannot be half agricultural and half mineral. It is a well-known
principle that the owner of piece of land has rights not only to its surface but also to everything underneath and the airspace above it up to a
reasonable height. Under the aforesaid ruling, the land is classified as mineral underneath and agricultural on the surface, subject to separate claims of
title. This is difficult to understand, especially in its practical application. What must have misled the respondent court is Commonwealth Act No. 137,
providing as follows:

Sec. 3. All mineral lands of the public domain and minerals belong to the State, and their disposition, exploitation, development or utilization, shall be
limited to citizens of the Philippines, or to corporations, or associations, at least 60% of the capital of which is owned by such citizens, subject to any
existing right, grant, lease or concession at the time of the inauguration of government established under the Constitution.

SEC. 4. The ownership of, and the right to the use of land for agricultural, industrial, commercial, residential, or for any purpose other than mining does
not include the ownership of, nor the right to extract or utilize, the minerals which may be found on or under the surface.

SEC. 5. The ownership of, and the right to extract and utilize, the minerals included within all areas for which public agricultural land patents are
granted are excluded and excepted from all such patents. This is an application of the Regalian doctrine which, as its name implies, is intended for the
benefit of the State, not of private persons. The rule simply reserves to the State all minerals that may be found in public and even private land devoted
to "agricultural, industrial, commercial, residential or (for) any purpose other than mining." Thus, if a person is the owner of agricultural land in which
minerals are discovered, his ownership of such land does not give him the right to extract or utilize the said minerals without the permission of the State
to which such minerals belong.

The flaw in the reasoning of the respondent court is in supposing that the rights over the land could be used for both mining and non-mining purposes
simultaneously. The correct interpretation is that once minerals are discovered in the land, whatever the use to which it is being devoted at the time,
such use may be discontinued by the State to enable it to extract the minerals therein in the exercise of its sovereign prerogative. The land is thus
converted to mineral land and may not be used by any private party, including the registered owner thereof, for any other purpose that will impede the
mining operations to be undertaken therein, For the loss sustained by such owner, he is of course entitled to just compensation under the Mining Laws
or in appropriate expropriation proceedings.

In the instant case, as already observed, the land which was originally classified as forest land ceased to be so and became mineral and completely
mineral once the mining claims were perfected. Our holding is that Benguet and Atok have exclusive rights to the property in question by virtue of
their respective mining claims which they validly acquired before the Constitution of 1935. The land was not and could not have been transferred to the
private respondents by virtue of acquisitive prescription, nor could its use be shared simultaneously by them and the mining companies for agricultural
and mineral purposes.

MARIANO RIOSA vs. CLARO VERZOSA and CIRIACO BULAN


G.R. No. L-7726
November 6, 1913

Facts: Plaintiff commenced an action against the defendants, the purpose of which was to secure an injunction against the defendants to prevent them
from harvesting and destroying the growing hemp upon a certain parcel of land and to require the defendants to deposit the hemp already harvested
with the deputy sheriff of the pueblo of Malinao. Upon the presentation of said petition, the Honorable Vicente Nepomuceno granted a temporary
injunction in conformity with the prayer of the petition. The defendants were duly served with a summons and a copy of the petition. The defendants
failed to appear and answer the complaint within the time prescribed by law and the rules of the court. By reason of the failure of the defendants to
appeal, the plaintiff presented a motion asking that a judgment by default be rendered against them, which motion was granted. Later, the honourable
judge, granted a permanent injunction against the defendants, restraining them from cultivating or harvesting the crops upon the said land or from
doing anything thereon which would tend to injure its value, and also found that the defendants had caused damages to the plaintiff in the sum of
P300. It appears from the record that the attorney for the defendants was present in court during the trial of the cause, but by reason of the fact that
the defendants had presented no answer, no defense whatever was made during the trial of the cause. The court held that If the defendant had
11
redeemed the land or should redeem the land in the time allowed by law, a motion then for relief from the judgment will be considered. At this juncture,
by reason of the ruling of Judge Moir upon the motion to dismiss, it becomes important to examine the particular facts, in order to fully understand
Judge Moir's conclusions.

In December 1909, a judgment was rendered against Claro Verzosa, for the sum of P320.87. Upon said judgment an execution was issued and was
levied upon the land described in the complaint. The said land was sold at public auction, under said execution, and was purchased by the plaintiff also
held a mortgage upon said land which was due on the 1st day of January, 1910. It will be remembered that the present action was commenced on the
25th of January, 1910. The record does not clearly disclose what action the plaintiff had taken to protect his interest in the land under his mortgage.
The present action was brought to protect the plaintiff's interest in the land, by virtue of his purchase of the same under said execution on the 1st of
December, 1909. It will be remembered that the law permits the owner of land which has been sold under an execution to redeem the same within a
period of twelve months.

Issue: What interest has the purchaser of the land sold under execution in the same during said twelve months? What right has he to interfere with the
owner in the management and control of such land? Is he entitled to the rents and profits during the twelve months? Can he eject the owner from the
possession of the same?

Ruling: We believe that the weight of authority is to the effect that the purchaser of lands sold at public auction under a writ of execution only has an
inchoate right in the property, subject to be defeated and terminated within a period of twelve months from the date of sale, by a redemption on the
part of the owner. In the present case the property was in the possession of the owner, and the inchoate right of the purchaser was subject to be
defeated at any moment during the period of redemption. The owner was entitled to remain in the possession of the land sold for the statutory term of
twelve months, and she might at any time defeat the inchoate right obtained by the purchaser by proper redemption within that period.

Under the law it would seem to be difficult to fully understand the right of the plaintiff to interfere in the manner in which he has attempted to interfere
with the owner of the land before the expiration of the twelve months within which the owner had a right to redeem the land.

That the defendants were in possession of the land at the time it was sold at public sale under execution on the 1st of December, 1909, and continued
in the possession of said land. 2. The plaintiff, by virtue of the purchase under the execution sale, acquired no right, except a mere inchoate right in the
land, until after the expiration of the period within which the defendants had right to redeem 3. The defendants redeemed the land in accordance with
the provisions of law within the legal period. It must follow, therefore, that the plaintiff was not entitled to the remedy prayed for in his petition and is,
therefore, not entitled to damages resulting from the use and occupation by the defendants. We find no reason in the record for reversing or modifying
the conclusions of the lower court. The judgment of the lower court is, therefore, hereby affirmed with the costs of this instance.

VELASCO v. ROSENBERG
G.R. No. 9921
October 26, 1915

FACTS: The present action was commenced in the CFI of Manila to recover from the defendant corporation the possession of a certain parcel of land,
together with the buildings thereon and the sum of P500 for each and every month from the 1 st of July, 1912, until the same is delivered to the plaintiff.
The plaintiff also petitioned for the appointment of a receiver to take charge and conserve the property in litigation during pendency of the action, which
receiver was appointed and took possession of the property. Defendant denied that plaintiff is entitled to the possession of the property in question.
Defendant also alleged that receiver not only took possession of property, but other property as well and that the plaintiff, through the receiver,
fraudulently used the name of defendant (Rosenberg) and solicits business from the public by the use of such name. Thus, defendant claims damage
from the appointment of the receiver, together with other damages. Upon presentation of the issue for trial and after hearing evidence, the judge found
that the plaintiff occupied the property in question, through its receiver, and used the trade name of the defendant, that the latter was damaged in the
sum of P500.

ISSUES:
1. The right of the purchaser at an execution sale to the rents and profits of the property sold when the execution debtor is in possession of the
same at the time of the sale, during the period of redemption, or for a period of one year thereafter; and
2. The right of the defendant to recover of the plaintiff damages resulting from continuance of the business sold under the execution, by a
receiver duly appointed by the court

HELD:
1. An analogous case decided by the SC, it held, insofar as the right of the purchaser is concerned to collect rent for the property during the
period of redemption when the execution debtor is in possession of the property, "that, inasmuch as, under the law, the rents received by the
purchaser during the period allowed for redemption must be applied on account of the redemption price, the judgment debtor in possession
of such property should not be required to pay rent, inasmuch as he would thereby simply be paying rent to himself. Hence, a judgment
debtor in possession of real property sold under execution cannot be required to pay rent to the purchaser for such property during the period
of redemption.
2. With reference to the damage caused by the receiver in continuing the business, it may be said that inasmuch as the receiver did continue to
run the business and inasmuch as he was an officer of the court, appointed thereby for the purpose of conserving the property we have a
right to assume that he was authorized so to do. The question of damages arising from his running the business should have been settled in
his final accounting to the court. If in such final accounting the receiver had not properly reported the same or accounted therefor, an
objection might properly have then been made to the accounting of the receiver and he held responsible therefor in case of loss through
negligence or by bad administration of the property given into his care. There is nothing in the record which shows that the property had not
been properly accounted for.

Sarmiento vs Agana
GR No. L-57288
April 30, 1984

12
Facts: While ERNESTO was still courting his wife, the latter's mother had told him the couple could build a RESIDENTIAL HOUSE on a lot of 145 sq.
ms., being Lot D of a subdivision in Paranaque (the LAND, for short). In 1967, ERNESTO did construct a RESIDENTIAL HOUSE on the LAND at a cost of
P8,000.00 to P10,000.00. It was probably assumed that the wife's mother was the owner of the LAND and that, eventually, it would somehow be
transferred to the spouses. It subsequently turned out that the LAND had been titled in the name of Mr. & Mrs. Jose C. Santo, Jr. who, on September
7,1974, sold the same to petitioner SARMIENTO. On January 6, 1975, SARMIENTO asked ERNESTO and wife to vacate and, on April 21, 1975, filed an
Ejectment suit against them. In the evidentiary hearings before the Municipal Court, SARMIENTO submitted the deed of sale of the LAND in her favor,
which showed the price to be P15,000.00. On the other hand, ERNESTO testified that the then cost of the RESIDENTIAL HOUSE would be from
P30,000.00 to P40,000.00. The figures were not questioned by SARMIENTO. Ejectment suit was elevated to the Court of First Instance of Pasay. It
ruled SARMIENTO was required, within 60 days, to exercise the option to reimburse ERNESTO and wife the sum of 40,000.00 as the value of the
RESIDENTIAL HOUSE, or the option to allow them to purchase the LAND for P25,000.00. SARMIENTO did not exercise any of the two options within the
indicated period, and ERNESTO was then allowed to deposit the sum of P25,000.00 with the Court as the purchase price for the LAND.

Issue: What are the rights of the parties if an accession is made with respect to Immovable property?

Held: ART. 448. The owner of the land on which anything has been built, sown or planted in good faith, shall have the right to appropriate as his
own the works, sowing or planting, after payment of the indemnity provided for in articles 546 and 548, OR to oblige the one who
built or planted to pay the price of the land, and the one who sowed, the proper rent. However, the builder or planter cannot be obliged to
buy the land if its value is considerably more than that of the building or trees. In such case, he shall pay reasonable rent, if the owner of the land does
not choose to appropriate the building or trees after proper indemnity. The parties shall agree upon the terms of the lease and in case of disagreement,
the court shall fix the terms thereof. The provision for the exercise by petitioner SARMIENTO of either the option to indemnify private respondents in
the amount of P40,000.00, or the option to allow private respondents to purchase the LAND at P25,000.00, in our opinion, was a correct decision.

BULACANAG vs. JUDGE FRANCISCO


G.R. No. L-34199
May 30, 1983

FACTS: Charvet and Stohner entered into a lease contract whereby the former is the lessor and the latter is the lessee. Later on, Stohner constructed a
house in the property. Much later on, Mrs. Charvet sold the subject property to Mr. Bulacanag. When Stohner's failed to pay the rents, Balucanag, thru
counsel, wrote Stohner a letter demanding that he vacate the premises. However, Stohner proposed that Bulacanag should reimbursed him the value of
the house first before he do so. As no agreement was reached, Balucanag instituted an ejectment suit against Stohner.

ISSUE: WON Stohner can be ejected until he is reimbursed of the value of the improvements?

RULING

Article 448 of the Civil Code, relied upon by respondent judge, applies only to a case where one builds on land in the belief that he is the owner thereof
and it does not apply where one's only interest in the land is that of a lessee under a rental contract. It cannot apply to a lessee because, as such
lessee, he knows that he is not the owner of the leased premises. The law applicable to the case at bar is Article 1678 of the Civil Code. This article
gives the lessor the option to appropriate the useful improvements by paying one-half of their value. However, the lessee cannot compel the lessor to
appropriate the improvements and make reimbursement for the lessee's right under the law is to remove the improvements even if the leased premises
may suffer damage thereby to the extent that he shall not cause any more damage upon the property than is necessary.

PNB v. DE JESUS
GR No. 149295
September 23, 2003

FACTS: In 1995, respondent Generoso filed a complaint against petitioner before the RTC of Occidental Mindoro for recovery of ownership and
possession, with damages, over the questioned property. In his complaint, respondent stated that he had acquired a parcel of land situated in
Mamburao, Occidental Mindoro and that on 26 March 1993, he had caused a verification survey of the property and discovered that the northern portion
of the lot was being encroached upon by a building of petitioner. Despite two letters of demand sent by respondent, petitioner failed and refused to
vacate the area. Petitioner, in its answer, asserted that when it acquired the lot and the building sometime in 198
1from then Mayor Bienvenido Ignacio, theencroachment already was in existence and toremedy the situation, Mayor Ignacio offered to sell the area in
question (which then also belonged to Ignacio) to petitioner at P100.00 per square meter which offer the latter claimed to have accepted. The sale,
however, did not materialize when, without the knowledge and consent of petitioner, Mayor Ignacio later mortgaged the lot to the Development Bank
of the Philippines. He also contends that he is a builder in good faith. The trial court decided the case in favor of respondent declaring him to be the
rightful owner of the disputed 124-square-meter portion of the lot and ordering petitioner to surrender possession of the property to respondent and to
cause, at its expense, the removal of any improvement thereon.

ISSUE: Whether petitioner is a builder in good faith and whether or not Art. 448 should be applied in favor of petitioner

HELD:
A builder in good faith is one who, not being the owner of the land, builds on that land believing himself to be its owner and unaware of any defect in
his title or mode of acquisition.

Article 448. The owner of the land on which anything has been built, sown, or planted in good faith, shall have the right to appropriate as his own the
works, sowing or planting, after payment of the indemnity provided for in Articles 546 and 548, or to oblige the one who built or planted to pay the
price of the land, and the one who sowed, the proper rent. However, the builder or planter cannot be obliged to buy the land if its value is considerably
more than that of the building or trees. In such a case, he shall pay reasonable rent, if the owner of the land does not choose to appropriate the
building or trees after proper indemnity. The parties shall agree upon the terms of the lease and in case of disagreement, the court shall fix the terms
thereof.

A builder in good faith can compel the landowner to make a choice between appropriating the building by paying the proper indemnity or obliging the
builder to pay the price of the land. The choice belongs to the owner of the land, a rule that accords with the principle of accession, i.e., that the
13
accessory follows the principal and not the other way around. Even as the option lies with the landowner, the grant to him is preclusive. He cannot, for
instance, compel the owner of the building to instead remove it from the land. In order, however, that the builder can invoke that accruing benefit and
enjoy his corresponding right to demand that a choice be made by the landowner, he should be able to prove good faith on his part.

Good faith is an intangible and abstract quality with no technical meaning or statutory definition, and it encompasses an honest belief, the absence of
malice and the absence of design to defraud or to seek an unconscionable advantage. Applied to possession, one is considered in good faith if he is not
aware that there exists in his title or mode of acquisition any flaw which invalidates it. Petitioner was not in good faith. Evidently, petitioner was quite
aware prior to its acquisition of the land and building from Ignacio that a part of the building sold to it stood on the land not covered by the land
conveyed to it. The building, constructed on the land by Ignacio, has in actuality been part of the property transferred to petitioner. Article 448, of the
Civil Code refers to a piece of land whose ownership is claimed by two or more parties, one of whom has built some works and not to a case where the
owner of the land is the builder, sower, or planter who then later loses ownership of the land by sale or otherwise for, elsewise stated, where the true
owner himself is the builder of works on his own land, the issue of good faith or bad faith is entirely irrelevant. In fine, petitioner is not in a valid
position to invoke the provisions of Article 448 of the Civil Code.

Heirs of Victorino Sarili vs Lagrosa


GR No. 193517
January 15, 2014

Facts: Lagrosa represented by Attorney-in-Fact, Atty. Lourdes Mojica, filed a complaint against Sps. Sarili alleging that he is owner of a land in
Caloocan City covered by TCT No. 55979. Respondent claimed that he is a resident of California, USA, and that during his vacation in the Philippines, he
discovered that a new certificate of title to the subject property was issued by the RD in the name of Victorino married to Isabel Amparo (Isabel), i.e.,
TCT No. 262218, by virtue of a falsified Deed of Absolute Sale9 dated February 16, 1978 (February 16, 1978 deed of sale) purportedly executed by him
and his wife, Amelia U. Lagrosa (Amelia). In their answer, Sps. Sarili maintained that they are innocent purchasers for value, having purchased the
subject property from Ramon B. Rodriguez (Ramon), who possessed and presented a Special Power of Attorney(subject SPA) to sell/dispose of the
same, and, in such capacity, executed a Deed of Absolute Sale dated November 20, 1992 conveying the said property in their favour. Sarili built a
house on the disputed land.

Issue: Whether or not the Sps. Sarili are builders in good faith entitled to reimbursement

Held: The strength of the buyers inquiry on the sellers capacity or legal authority to sell depends on the proof of capacity of the seller. If the proof of
capacity consists of a special power of attorney but there appears to be flaws in its notarial acknowledgment, mere inspection of the document will not
do; the buyer must show that his investigation went beyond the document and into the circumstances of its execution.

In the present case, it is undisputed that Sps. Sarili purchased the subject property from Ramos on the strength of the latters ostensible authority to
sell under the subject SPA. The said document, however, readily indicates flaws in its notarial acknowledgment since the respondents community tax
certificate (CTC) number was not indicated thereon. Despite this irregularity, however, Sps. Sarili failed to show that they conducted an investigation
beyond the subject SPA and into the circumstances of its execution as required by prevailing jurisprudence. Hence, Sps. Sarili cannot be considered
as innocent purchasers for value.

ART. 450. The owner of the land on which anything has been built, planted or sown in bad faith may demand the demolition of the work, or that the
planting or sowing be removed, in order to replace things in their former condition at the expense of the person who built, planted or sowed; or he may
compel the builder or planter to pay the price of the land, and the sower the proper rent.

As for Sps. Sarili, they knew or at the very least, should have known from the very beginning that they were dealing with a person who possibly had
no authority to sell the subject property considering the palpable irregularity in the subject SPAs acknowledgment. Yet, relying solely on said document
and without any further investigation on Ramoss capacity to sell Sps. Sarili still chose to proceed with its purchase and even built a house thereon.
Based on the foregoing it cannot be seriously doubted that Sps. Sarili were actually aware of a flaw or defect in their title or mode of acquisition and
have consequently built the house on the subject property in bad faith under legal contemplation. The case is therefore remanded to the court a quo for
the proper application of the above-cited Civil Code provisions.

Communities Cagayan Inc. Vs. Spouses Arsenio and Angeles Nanol


G.R. NO. 176791
Nov. 14, 2012

Facts: Respondent Spouses Arsenio and Angeles Nanol entered into a contract to sell with petitioner Communities Cagayan Inc. wherein the latter
agreed to sell the House And lot of lot 17 and 19 at block 16 for 368,00php. Respondent-spouses, however, did not avail of petitioners in house
Financing due to its high interest rates. Petitioner use the property as collateral for the loan. To facilitate the loan, a simulated sale over the property
was executed by petitioner in favor of respondent-spouses. Unfortunately, the bank collapsed and closed before it could release the loan. Thus, on
November 30, 1997, respondent-spouses entered into another Contract to Sell with petitioner over the same property for the same price of
P368,000.00. This time, respondent-spouses availed of petitioners in-house
financing thus, undertaking to pay the loan over four years, from 1997 to 2001. On September 10, 2003, petitioner sent respondent-spouses a
notarized Notice of Delinquency and Cancellation of Contract to Sell due to the latters failure to pay the monthly amortizations. Petitioner filed for
complaint of cancellation of title and recovery of possession, reconveyance and damages in the RTC. It ruled that the sale is void and the transfer of
title to respondent is cancelled. Instead of appealing, Petitioner opted to file the instant petition directly with this Court on a pure question of law .

Issues:
1. Whether petitioner is obliged to refund to respondent-spouses all the monthly installments paid.
2. Whether petitioner is obliged to reimburse respondent-spouses the value of the new house minus the cost of the original house.

Ruling:
1. Considering that this case stemmed from a Contract to Sell executed by the petitioner and the respondent-spouses, we agree with petitioner
that the Maceda Law, which governs sales of real estate on installment, should be applied. (b) If the contract is canceled, the seller
14
shall refund to the buyer the cash surrender value of the payments on the property equivalent to fifty percent of the total
payments made, and, after five years of installments, an additional five per cent every year but not to exceed ninety per cent of the total
payments made: Provided, That the actual cancellation of the contract shall take place after thirty days from receipt by the buyer of the notice
of cancellation or the demand for rescission of the contract by a notarial act and upon full payment of the cash surrender value to the buyer.
In this connection, we deem it necessary to point out that, under the Maceda Law, the actual cancellation of a contract to sell takes place
after 30 days from receipt by the buyer of the notarized notice of cancellation, and upon full payment of the cash surrender value to the
buyer. In other words, before a contract to sell can be validly and effectively cancelled, the seller has (1) to send a notarized notice of
cancellation to the buyer and (2) to refund the cash surrender value.
2. In Tuatis, we ruled that the seller (the owner of the land) has two options under Article 448: (1) he may appropriate the improvements for
himself after reimbursing the buyer (the builder in good faith) the necessary and useful expenses under Articles 546 and 548 of the Civil
Code; or (2) he may sell the land to the buyer, unless its value is considerably more than that of the improvements, in which case, the buyer
shall pay reasonable rent. In conformity with the foregoing pronouncement, we hold that petitioner, as landowner, has two options. It may
appropriate the new house by reimbursing respondent Angeles the current market value thereof minus the cost of the old house. Under this
option, respondent Angeles would have a right of retention which negates the obligation to pay rent. In the alternative, petitioner may sell
the lots to respondent Angeles at a price equivalent to the current fair value thereof. However, if the value of the lots is considerably more
than the value of the improvement, respondent Angeles cannot be compelled to purchase the lots. She can only be obliged to pay petitioner
reasonable rent. In view of the foregoing disquisition and in accordance with Depra v. and Technogas Philippines Manufacturing Corporation
v. Court of Appeals, we find it necessary to remand this case to the court of origin for the purpose of determining matters necessary for the
proper application of Article 448, in relation to Articles 546 and 548 of the Civil Code.

VDA DE ROXAS v. OUR LADYS FOUNDATION


GR No. 182378
March 6, 2013

FACTS: On September 1, 1988, Salve Dealca Latosa filed before the RTC a complaint for the recovery of ownership of a portion of her residential land
located at Our Ladys Village, Bibincahan, Sorgsogon. According to her, Roxas, represented by petitioner herein, encroached on a quarter of her
property by arbitrarily extending his concrete fence beyond the correct limits. In his answer, Roxas imputed the blame to respondent Our Ladys
Foundation, Inc. then filed a Third-Party complaint against respondent and claimed that he only occupied the adjoining portion to get the equivalent
area of what he lost when OFLI trimmed his property for the subdivision road. After considering the evidence of all the parties, the trial court held that
Latosa had established her claim of encroachment by a preponderance of evidence. It found that Roxas occupied a total of 112 square meters of
Latosas lots, and that, in turn, OLFI trimmed his property by 92 square meters. Thus, Roxas was ordered to return and surrender the portion of 116 sq.
meters which lawfully belongs to plaintiff and to demolish whatever structure constructed thereon. OFLI was ordered to reimburse Roxas the value of
the 92 sq. meters plus interest to be reckoned from the time it was paid to the third-party defendant.

ISSUE: Whether the valuation of the amount OFLI should reimburse Roxas reflect the current value of the property or the original amount of the lot

HELD: Both the trial and the appellate courts differed in interpreting the amount of reimbursement payable by respondent (OFLI) to petitioner (Roxas).
The RTC pegged the reimbursable amount at 1,800 per square meter to reflect the current value of the property, while the CA maintained the original
amount of the lot at 40 per square meter. To settle the contention, this Court resorts to the provisions of the Civil Code governing encroachment on
property. Under Article 448 pertaining to encroachments in good faith, as well as Article 450 referring to encroachments in bad faith, the owner of the
land encroached upon petitioner herein has the option to require respondent builder to pay the price of the land.

Although these provisions of the Civil Code do not explicitly state the reckoning period for valuing the property, jurisprudence states that in the event
that the seller elects to sell the lot, the price must be fixed at the prevailing market value at the time of payment. Jurisprudence states that the present
or current fair value of the land is to be reckoned at the time that the landowner elected the choice, and not at the time that the property was
purchased. In another case, the Court reckoned the valuation of the property at the time that the real owner of the land asked the builder to vacate the
property encroached upon. The CA incorrectly pegged the reimbursable amount at the old market value of the subject property 40 per square meter
as reflected in the Deed of Absolute Sale between the parties. The RTC properly considered in its 2 December 2004 Order the value of the lot at
1,800 per square meter, the current fair price.

Parilla vs Pilar
GR No. 167680
November 30, 2006

Facts: Petitioner-spouses Samuel and Chinita Parilla and their co-petitioner-son Deodato Parilla, as dealers of Pilipinas Shell Petroleum Corporation
(Pilipinas Shell), have been in possession of a parcel of land (the property) located at the poblacion of Bantay, Ilocos Sur which was leased to it by
respondent Dr. Prospero Pilar under a 10-year Lease Agreement entered into in 1990. When the lease contract between Pilipinas Shell and respondent
expired in 2000, petitioners remained in possession of the property on which they built improvements consisting of a billiard hall and a restaurant,
maintained a sari-sari store managed by Leonardo Dagdag, Josefina Dagdag and Edwin Pugal, and allowed Flor Pelayo, Freddie Bringas and Edwin
Pugal to use a portion thereof as parking lot.

Pilar thru Marivic Padre filed a complaint for ejectment in the MTC. MTC ruled in favour of the respondent ordering Parilla et al to vacate the premises
and at the same time it ordered Pilar to reimburse petitioners the sum of 2 million which represented the value of the improvements introduced in the
property. Pilar appealed to RTC but RTC affirmed the MTC. Pilar went to CA. CA set aside the ruling of the RTC. Hence, this present petition by Parilla.

Issue: Whether or not Article 448 governing the right of accession in relation to Article 546 pertaining to effects of possession is applicable in this case

Held: The law on lease under the New Civil Code has specific rules concerning useful improvements introduced by a lessee on the property leased, it is
erroneous on the part of petitioners to urge this Court to apply Article 448, in relation to Article 546, regarding their claim for reimbursement and to
invoke the right of retention before reimbursement is made. Jurisprudence is replete with cases which categorically declare that Article 448 covers only
cases in which the builders, sowers or planters believe themselves to be owners of the land or, at least, have a claim of title thereto, but not when the
interest is merely that of a holder, such as a mere tenant, agent or usufructuary. A tenant cannot be said to be a builder in good faith as he has no
pretension to be owner.
15
ART 1678 not ART 448 applies. Art. 1678. If the lessee makes, in good faith, useful improvements which are suitable to the use for which the lease is
intended, without altering the form or substance of the property leased, the lessor upon the termination of the lease shall pay the lessee one-half of the
value of the improvements at that time. Should the lessor refuse to reimburse said amount, the lessee may remove the improvements, even though the
principal thing may suffer damage thereby. He shall not, however, cause any more impairment upon the property leased than is necessary . Under
Article 1678, it is the lessor who is given the option, upon termination of the lease contract, either to appropriate the useful improvements by paying
one-half of their value at that time, or to allow the lessee to remove the improvements. This option solely belongs to the lessor as the law is explicit that
[s]hould the lessor refuse to reimburse said amount, the lessee may remove the improvements, even though the principal thing may suffer damage
thereby. It appears that the lessor has opted not to reimburse.

VERONA PADA-KILARIO and RICARDO KILARIO vs. COURT OF APPEALS and SILVERIO PADA
G.R. No. 134329
January 19, 2000

Facts: One Jacinto Pada had six (6) children, namely, Marciano, Ananias, Amador, Higino, Valentina and Ruperta. He died intestate. His estate included
a parcel of land of residential and coconut land located at Poblacion, Matalom, Leyte, denominated as Cadastral Lot No. 5581 with an area of 1,301.92
square meters. It is the northern portion of Cadastral Lot No. 5581 which is the subject of the instant controversy. During the lifetime of Jacinto Pada,
his half-brother, Feliciano Pada, obtained permission(take note mau ni tubag sa number 2 ruling) from him to build a houseon the northern
portion of Cadastral Lot No. 5581. When Feliciano died, his son, Pastor, continued living in the house together with his eight children. Petitioner Verona
Pada-Kilario, one of Pastor's children, has been living in that house since 1960. Sometime in May, 1951, the heirs of Jacinto Pada entered into an extra-
judicial partition of his estate. For this purpose, they executed a private document which they, however, never registered in the Office of the Registrar
of Deeds of Leyte(take note ani nga wala na register). On June 14, 1978, Juanita(anak ni ananias) Pada sold to Engr. Ernesto Paderes, the right of his
father, Ananias, as co-owner of Cadastral Lot No. 5881. On November 17, 1993, it was the turn of Maria Pada to sell the co-ownership right of his
father, Marciano. Private respondent, who is the first cousin of Maria, was the buyer.Thereafter, private respondent demanded that petitioner spouses
vacate the northern portion of Cadastral Lot No. 5581 so his family can utilize the said area. They went through a series of meetings with the barangay
officials concerned for the purpose of amicable settlement, but all earnest efforts toward that end, failed. The Municipal Circuit Trial Court rendered
judgment in favor of petitioner spouses. From the foregoing decision, private respondent appealed to the Regional Trial Court. On November 6, 1997, it
rendered a judgment of reversal. Petitioners filed in the Court of Appeals a petition for review of the foregoing decision of the Regional Trial Court. On
May 20, 1998, respondent Court of Appeals rendered judgment dismissing said petition.
Issue:
1. WHETHER THE COURT OF APPEALS ERRED IN NOT RULING THAT PETITIONERS, AS CO-OWNERS, CANNOT BE EJECTED FROM THE
PREMISES CONSIDERING THAT THE HEIRS OF JACINTO PADA DONATED TO THEM THEIR UNDIVIDED INTEREST IN THE PROPERTY IN
DISPUTE.
2. WHETHER OR NOT THE PETITIONERS ARE BUILDERS IN GOOD FAITH
Ruling:
1. We hold that the extrajudicial partition of the estate of Jacinto Pada among his heirs made in 1951 is valid, albeit executed in an unregistered
private document. No law requires partition among heirs to be in writing and be registered in order to be valid. The requirement
in Sec. 1, Rule 74 of the Revised Rules of Court that a partition be put in a public document and registered, has for its purpose the
protection of creditors and the heirs themselves against tardy claims. The object of registration is to serve as constructive notice to others.
It follows then that the intrinsic validity of partition not executed with the prescribed formalities is not undermined when no creditors are
involved. Without creditors to take into consideration, it is competent for the heirs of an estate to enter into an agreement for
distribution thereof in a manner and upon a plan different from those provided by the rules from which, in the first place, nothing can be
inferred that a writing or other formality is essential for the partition to be valid.
2. Petitioners are estopped from impugning the extrajudicial partition executed by the heirs of Jacinto Pada after explicitly admitting in their
Answer that they had been occupying the subject property since 1960 without ever paying any rental as they only relied on the liberality and
tolerance of the Pada family.25 Their admissions are evidence of a high order and bind them insofar as the character of their possession of the
subject property is concerned. Considering that petitioners were in possession of the subject property by sheer tolerance of its owners, they
knew that their occupation of the premises may be terminated any time. Persons who occupy the land of another at the latter's
tolerance or permission, without any contract between them, is necessarily bound by an implied promise that they will
vacate the same upon demand, failing in which a summary action for ejectment is the proper remedy against them.Thus, they
cannot be considered possessors nor builders in good faith. It is well-settled that both Article 448 and Article 546 of the New Civil Code which
allow full reimbursement of useful improvements and retention of the premises until reimbursement is made, apply only to a possessor in
good faith, i.e., one who builds on land with the belief that he is the owner thereof.
IGNACIO v. DIRECTOR OF LANDS
GR No. L-12958
May 30, 1960

FACTS: Faustino Ignacio filed an application to register a parcel of land (mangrove) which he alleged he acquired by right of accretion since it adjoins a
parcel of land owned by the Ignacio. His application is opposed by the Director of Lands, Laureano Valeriano, contending that said land forms part of
the public domain. The Trial Court dismissed the application holding that said land formed part of the public domain. Appellant contends that the parcel
belongs to him by the law of accretion, having been formed by gradual deposit by action of the Manila Bay, and he cites Article 457 of the New Civil
Code.

ISSUE: Whether or not the land subject of the dispute can be acquired by right of accretion of Ignacio.

HELD: No. Article 457 is not applicable. The article cited is clearly inapplicable because it refers to accretion or deposits on the banks of rivers, while the
accretion in the present case was caused by action of the Manila Bay. Manila bay is not a river. A bay is a part of the sea, being a mere indentation of
the same. A bay is an opening into the land where the water is shut in on all sides except at the entrance; an inlet of the sea; an arm of the sea,
distinct from a river, a bending or curbing of the shore of the sea or of a lake. The law on accretion cited by Ignacio in inapplicable in the present case
because it refers to accretion or deposits on the banks of rivers while this refers to action in the Manila Bay, which is held to be part of the sea.

16
De Buyser vs Director of Lands
GR No. L-22763
March 18, 1983

Facts: Plaintiff-appellant is the registered owner of Lot No. 4217 of the Surigao Cadastre, which borders the Surigao Strait. Contiguous to said lot is a
parcel of land which was formed by accretion from the sea, the subject- matter of this controversy. Defendants Ignacio Tandayag and his wife Candida
Tandayag have been occupying this foreshore land under a Revocable Permit issued by the Director of Lands. Claiming ownership of the said land,
plaintiff filed an action against the spouses Tandayag in the Court of First Instance of Surigao to recover possession of this land as well as rents in
arrears for a period of six years. In asserting the right of ownership over the land, plaintiff invokes Article 4 of the Spanish Law of Waters of August 3,
1866 which provides: Art. 4. Lands added to the shore by accretion and alluvial deposits caused by the action of the sea, form part of the public
domain, when they are no longer washed by the waters of the sea, and are not necessary for purposes of public utility, or for the establishment of
special industries, or for the coastguard service, the Government shall declare them to be the property of the owners of the estate adjacent thereto and
as an increment thereof.

Issue: Whether or not De Buyser can validly claim ownership of such land formed by accretion from the sea

Held: Plaintiff's reliance on the above article is quite misplaced. The true construction of the cited provision is that the State shall grant these lands to
the adjoining owners only when they are no longer needed for the purposes mentioned therein. In the case at bar, the trial court found that plaintiff's
evidence failed to prove that the land in question is no longer needed by the government, or that the essential conditions for such grant under Article 4
of the Spanish Law of Waters exists. Plaintiff, however, argues that the approval by the Director of Lands of the defendants' Revocable Permit
Application is tantamount to an implied declaration on the part of the Director of Lands of the fact that the disputed lot is no longer needed for public
use. We fail to see such implication. The Director of Lands wrote a letter approving the Revocable Permit Application. It did not declare the land as no
longer needed for public use. It says
land applied for by you is/may be needed by the Government for future public improvements (Boulevard and seawall protection purposes) you may be
allowed to continue with your temporary occupation and provisional use of the premises under a revocable permit renewable every year in the
meantime that the land is not actually needed by the Government for the purposes aforestated

Requisites of Alluvion: Accretion and Alluvial Deposits must be caused by the action of the sea; It is no longer washed by the waters of the sea; The
Government must declare that it is no longer necessary for purposes of public utility, or for establishment of special industries, or for coastguard
services

REPUBLIC OF THE PHILIPPINES (DIRECTOR OF LANDS) vs. THE HON. COURT OF APPEALS, BENJAMIN TANCINCO, AZUCENA
TANCINCO REYES, MARINA TANCINCO IMPERIAL and MARIO C. TANCINCO
G.R. No. L-61647
Oct. 12, 1984

Facts: Respondents Benjamin Tancinco, Azucena Tancinco Reyes, Marina Tancinco Imperial and Mario C. Tancinco are registered owners of a parcel of
land covered by Transfer Certificate of Title No. T-89709 situated at Barrio Ubihan, Meycauayan, Bulacan bordering on the Meycauayan and Bocaue
rivers. On June 24, 1973, the private respondents filed an application for the registration of three lots adjacent to their fishpond property. On April 5,
1974, Assistant Provincial Fiscal Amando C. Vicente, in representation of the Bureau of Lands filed a written opposition to the application for registration.
On March 6, 1975, the private respondents filed a partial withdrawal of the application for registration with respect to Lot 3 of Plan Psu-131892 in line
with the recommendation of the Commissioner appointed by the Court. On March 7, 1975, Lot 3 was ordered withdrawn from the application and trial
proceeded only with respect to Lots 1 and 2 covered by Plan Psu-131892. On June 26, 1976, the lower court rendered a decision granting the
application on the finding that the lands in question are accretions to the private respondents' fishponds covered by Transfer Certificate of Title No.
89709. Only Lot 1 and 2 where granted. The petitioner submits that there is no accretion to speak of under Article 457 of the New Civil Code because
what actually happened is that the private respondents simply transferred their dikes further down the river bed of the Meycauayan River, and thus, if
there is any accretion to speak of, it is man-made and artificial and not the result of the gradual and imperceptible sedimentation by the waters of the
river.
Issue: WON there is actual accretion on the subject Lots.
Ruling: The court agrees with the petitioner. Article 457 of the New Civil Code provides: To the owners of lands adjoining the banks of rivers belong
the accretion which they gradually receive from the effects of the current of the waters. The article requires the concurrence of three requisites before
an accretion covered by this particular provision is said to have taken place. They are (1) that the deposit be gradual and imperceptible; (2) that it be
made through the effects of the current of the water; and (3) that the land where accretion takes place is adjacent to the banks of rivers.
The private respondents rely on the testimony of Mrs. Virginia Acua to the effect that: when witness first saw the land, namely, Lots 1 & 2, they were
already dry almost at the level of the Pilapil of the property of Dr. Tancinco, and that from the boundaries of the lots, for about two (2) arms length the
land was still dry up to the edge of the river; that sometime in 1951, a new Pilapil was established on the boundaries of Lots 1 & 2 and soil from the old
Pilapil was transferred to the new Pilapil and this was done sometime in 1951; that the new lots were then converted into fishpond, and water in this
fishpond was two (2) meters deep on the side of the Pilapil facing the fishpond.
The private respondents submit that the foregoing evidence establishes the fact of accretion without human intervention because the transfer of the
dike occurred after the accretion was complete. The requirement that the deposit should be due to the effect of the current of the river is indispensable.
This excludes from Art. 457 of the New Civil Code all deposits caused by human intervention. Alluvion must be the exclusive work of nature. In the
instant case, there is no evidence whatsoever to prove that the addition to the said property was made gradually through the effects of the current of
the Meycauayan and Bocaue rivers. We agree with the observation of the Solicitor General that it is preposterous to believe that almost four (4)
hectares of land came into being because of the effects of the Meycauayan and Bocaue river.
REPUBLIC v. ARCADIO IVAN SANTOS
GR No. 160453
November 12, 2012

By law, accretion - the gradual and imperceptible deposit made through the effects of the current of the water- belongs to the owner of the land
adjacent to the banks of rivers where it forms. The drying up of the river is not accretion. Hence, the dried-up river bed belongs to the State as property
of public dominion, not to the riparian owner, unless a law vests the ownership in some other person.

17
FACTS: Alleging continuous and adverse possession of more than ten years, respondent Arcadio Ivan applied on March 7, 1997 for the registration of
Lot 4998-B (the property) in the RTC. On May 21, 1998, Arcadio Ivan amended his application for land registration to include Arcadio, Jr. as his co-
applicant because of the latters co-ownership of the property. He alleged that the property had been formed through accretion and had been in their
joint open, notorious, public, continuous and adverse possession for more than 30 years. The City of Paraque (the City) opposed the application for land
registration, stating that it needed the property for its flood control program; that the property was within the legal easement of 20 meters from the
river bank; and that assuming that the property was not covered by the legal easement, title to the property could not be registered in favor of the
applicants for the reason that the property was an orchard that had dried up and had not resulted from accretion. On May 10, 2000, the RTC granted
the application for registration. The Republic, through the OSG, appealed. The CA affirmed the RTC. Article 457 of the Civil Code provides that "(t)o the
owners of lands adjoining the banks of rivers belong theaccretion which they gradually receive from the effects of the currents of the waters. In rulings
for the respondents, the CA upheld the RTC which pronounced that on the basis of the evidence presented by the applicants, the court finds that the
respondents are the owners of the land subject of the application which was previously a part of the river which became an orchard after it dried up and
considering the lot which adjoins the same property is owned by the applicant which was obtained by the latter from his mother.

ISSUE: Whether or not respondents could claim the property by virtue of acquisitive prescription (section 14(1) of PD 1529).

HELD: No. The CA grossly erred in applying Article 457 of the Civil Code to respondents benefit. Article 457 of the Civil Code provides that "(t)o the
owners of lands adjoining the banks of rivers belong the accretion which they gradually receive from the effects of the currents of the waters."
Respondents as the applicants for land registration carried the burden of proof to establish the merits of their application by a preponderance of
evidence, by which is meant such evidence that is of greater weight, or more convincing than that offered in opposition to it. They would be held
entitled to claim the property as their own and apply for its registration under the Torrens system only if they established that, indeed, the property was
an accretion to their land. Accretion is the process whereby the soil is deposited along the banks of rivers. The deposit of soil, to be considered
accretion, must be: (a) gradual and imperceptible; (b) made through the effects of the current of the water; and (c) taking place on land adjacent to
the banks of rivers.

Accordingly, respondents should establish the concurrence of the elements of accretion to warrant the grant of their application for land registration.
However, respondents did not discharge their burden of proof. The RTC and the CA grossly erred in treating the dried-up river bed as an accretion that
became respondents property pursuant to Article 457 of the Civil Code. That land was definitely not an accretion. The process of drying up of a river to
form dry land involved the recession of the water level from the river banks, and the dried-up land did not equate to accretion, which was the gradual
and imperceptible deposition of soil on the river banks through the effects of the current. In accretion, the water level did not recede and was more or
less maintained. Hence, respondents as the riparian owners had no legal right to claim ownership of Lot 4998-B. Considering that the clear and
categorical language of Article 457 of the Civil Code has confined the provision only to accretion, we should apply the provision as its clear and
categorical language tells us to. Axiomatic it is, indeed, that where the language of the law is clear and categorical, there is no room for interpretation;
there is only room for application.

Bagaipo vs Court of Appeals


GR No. 116290
December 8, 2000

Facts: Petitioner Dionisia P. Bagaipo is the registered owner of Lot No. 415, a 146,900 square meter agricultural land situated in Ma-a, Davao City. In
the southeast portion of the said land was the Davao River. Respondent Leonor Lozano is the owner of a registered parcel of land located across and
opposite the southeast portion of petitioners lot facing the Davao River. On May 26, 1989, Bagaipo filed a complaint for Recovery of Possession with
Mandatory Writ of Preliminary Injunction and Damages against Lozano. She sought the recovery of a land area measuring 37,901 square meters which
Bagaipo allegedly lost when the Davao River traversed her property. Bagaipo contended that as a result of a change in course of the said river, her
property became divided into three lots, namely: Lots 415-A, 415-B and 415-C. For his part, Lozano insisted that the land claimed by Bagaipo is actually
an accretion to their titled property. He asserted that the Davao River did not change its course and that the reduction in Bagaipos domain was caused
by gradual erosion due to the current of the Davao River. On April 5, 1991, the trial court conducted an ocular inspection. It concluded that the
applicable law is Article 457 To the owners of lands adjoining the banks of rivers belong the accretion which they gradually receive from the effects of
the current of the waters of the New Civil Code and not Art. 461 The reduction in the land area of plaintiff was caused by erosion and not by a change
in course of the Davao River. Conformably then, the trial court dismissed the complaint. Bagaipo appealed to the CA. It affirmed the RTC.

Issue: Is Article 461 applicable in this case?

Held: Art. 461 of the Civil Code is inapplicable. Petitioner cannot claim ownership over the old abandoned riverbed because the same is inexistent. The
riverbeds former location cannot even be pinpointed with particularity since the movement of the Davao River took place gradually over an unspecified
period of time, up to the present. The rule is well-settled that accretion benefits a riparian owner when the following requisites are present: 1. That the
deposit be gradual and imperceptible; 2. That it resulted from the effects of the current of the water; and 3. That the land where accretion takes place
is adjacent to the bank of the river. These requisites were sufficiently proven in favor of respondents. In the absence of evidence that the change in the
course of the river was sudden or that it occurred through avulsion, the presumption is that the change was gradual and was caused by alluvium and
erosion.

MARIO C. RONQUILLO vs. THE COURT OF APPEALS, DIRECTOR OF LANDS, DEVELOPMENT BANK OF THE PHILIPPINES, ROSENDO DEL
ROSARIO, AMPARO DEL ROSARIO and FLORENCIA DEL ROSARIO
G.R. No. L-43346
March 20, 1991

Facts: It appears that plaintiff Rosendo del Rosario was a registered owner of a parcel of land known as Lot 34,Block 9, Sulucan Subdivision, situated at
Sampaloc, Manila and covered by Transfer Certificate of Title No. 34797 of the Registry of Deeds of Manila. The other plaintiffs Florencia and Amparo
del Rosario were daughters of said Rosendo del Rosario. Adjoining said lot is a dried-up portion of the old Estero Calubcub occupied by the defendant
since 1945 which is the subject matter of the present action. Rosario, the latter had been in possession of said lot including the adjoining dried-up
portion of the old Estero Calubcub having bought the same from Arsenio Arzaga. Sometime in 1935, said titled lot was occupied by Isabel Roldan with
the tolerance and consent of the plaintiff on condition that the former will make improvements on the adjoining dried-up portion of the Estero Calubcub.
In the early part of 1945 defendant occupied the eastern portion of said titled lot as well as the dried-up portion of the old Estero Calubcub which abuts
18
plaintiffs' titled lot. After a relocation survey of the land in question sometime in 1960, plaintiffs learned that defendant was occupying a portion of their
land and thus demanded defendant to vacate said land when the latter refused to pay the reasonable rent for its occupancy. However, despite said
demand defendant refused to vacate. On December 26, 1962, the trial court rendered judgment in favor of the Rosarios. Upon motion of Ronquillo,
respondent court modified its decision by setting aside the first portion of the trial court's decision ordering Ronquillo to surrender to the Del Rosarios
that portion of land covered by Transfer Certificate of Title No. 34797 occupied by the former, based on the former's representation that he had already
vacated the same prior to the commencement of this case. However, respondent court upheld its declaration that the Del Rosarios are the rightful
owners of the dried-up river bed. Hence, this petition.
On May 17, 1976, this Court issued a resolution 7 requiring the Solicitor General to comment on the petition in behalf of the Director of Lands as an
indispensable party in representation of the Republic of the Philippines, and who, not having been impleaded, was subsequently considered impleaded
as such in our resolution of September 10, 1976. In his Motion to Admit Comment, 9 the Solicitor General manifested that pursuant to a request made
by this office with the Bureau of Lands to conduct an investigation, the Chief of the Legal Division of the Bureau sent a communication informing him
that the records of his office "do not show that Mario Ronquillo, Rosendo del Rosario, Amparo del Rosario or Florencia del Rosario has filed any public
land application covering parcels of land situated at Estero Calubcub Manila as verified by our Records DivisionIn a letter dated June 29, 1979 . Florencia
del Rosario manifested to this Court that Rosendo, Amparo and Casiano del Rosario have all died, and that she is the only one still alive among the
private respondents in this case.
In a resolution dated January 20, 1988, the Court required petitioner Ronquillo to implead one Benjamin Diaz pursuant to the former's
manifestation that the land adjacent to the dried up river bed has already been sold to the latter, and the Solicitor General was also required to inquire
into the status of the investigation being conducted by the Bureau of LandsOn April 3, 1989, petitioner filed an Amended Petition for Certiorari, this
time impleading the Development Bank of the Philippines (DBP) which subsequently bought the property adjacent to the dried-up river bed from
Benjamin Diaz. In its resolution dated January 10, 1990, the Court ordered that DBP be impleaded as a party respondent
Issue: Whether the dried-up portion of Estero Calubcub being claimed by herein petitioner was caused by a natural change in the course of the waters;
and, corollary thereto, is the issue of the applicability of Article 370 of the old Civil Code.
Ruling:
Respondent court, in affirming the findings of the trial court that there was a natural change in the course of Estero Calubcub declared that:The
defendant claims that Article 370 of the old Civil Code is not applicable to the instant case because said Estero Calubcub did not actually change its
course but simply dried up, hence, the land in dispute is a land of public domain and subject to the disposition of the Director of Land(s). The contention
of defendant is without merit. As mentioned earlier, said estero as shown by the relocation plan did not disappear but merely changed its course by a
more southeasternly direction. As such, "the abandoned river bed belongs to the plaintiffs-appellees and said land is private and not public in nature.
Hence, further, it is not subject to a Homestead Application by the appellant". Even assuming for the sake of argument that said estero did not change
its course but merely dried up or disappeared, said dried-up estero would still belong to the riparian owner. A careful perusal of the evidence presented
by both parties in the case at bar will reveal that the change in the course of Estero Calubcub was caused, not by natural forces, but due to the
dumping of garbage therein by the people of the surrounding neighborhood. Under the circumstances, a review of the findings of fact of respondent
court thus becomes imperative. The foregoing facts and circumstances remove the instant case from the applicability of Article 370 of the old Civil Code
which provides:
Art. 370. The beds of rivers, which are abandoned because of a natural change in the course of the waters, belong to the owners of the
riparian lands throughout the respective length of each. If the abandoned bed divided tenements belonging to different owners the new
dividing line shall be equidistant from one and the other.
The law is clear and unambiguous. It leaves no room for interpretation.Article 370 applies only if there is a natural change in the course of the waters.
The rules on alluvion do not apply to man-made or artificial accretions nor to accretions to lands that adjoin canals or esteros or artificial drainage
systems. Considering our earlier finding that the dried-up portion of Estero Calubcub was actually caused by the active intervention of man, it follows
that Article 370 does not apply to the case at bar and, hence, the Del Rosarios cannot be entitled thereto supposedly as riparian owners. The dried-up
portion of Estero Calubcub should thus be considered as forming part of the land of the public domain which cannot be subject to acquisition by private
ownership.
SIARY VALLEY ESTATES, INC. v. LUCASAN
97 Phil 987
1955

FACTS: Siari Valley Inc. brought action to recover 200 heads of cattle that were driven from its lands to that of Lucasans. Lucasan however argued that
although there was commixtion of cattle, Siari already retrieved its animals. The CFI of Zamboanga decided in favor of Siari thus the case at bar.

ISSUE: Whether or not Lucasan was in bad faith thus should lose his share in the commixtion

HELD: YES. Although there was no actual evidence that all 823 missing animals were taken by Lucasan or his men, on 2 occasions however, his men
drove away 30 heads of cattle. It is not erroneous to believe that the others must have also been driven away applying by analogy the principle that one
who stole a part of the stolen money must have taken also the larger sum lost by the offended party. Art. 382 (now Art. 473) of the CC states that if
the commingling of 2 things is made in bad faith, the one responsible for it will lose his share thus since Lucasan is in bad faith, he should lose his
share in the commixtion. The SC ordered Lucasan to deliver the 321 heads that had been entrusted to his care to Siari; pay damages for the 400 heads
he sold since 1946; ordered to allow Siari to round up all the buffaloes that may be found on its cattle ranch.

19

Das könnte Ihnen auch gefallen